Class 8 Geography Chapter 6 Extra Questions and Answers Human Resource

Class 8 Geography Chapter 6 Extra Questions and Answers Human Resource

Get ready to embark on an enlightening expedition through the annals of human resource, where the echoes of human potential resonate with the spirit of inclusive development. Together, we’ll explore the profound insights of “Class 8 Geography Chapter 6 Extra Questions and Answers Human Resource” and gain a deeper appreciation for the transformative power of human capital in shaping our world. Read this also Extra Questions for Class 8 Social Science .

Class 8 Geography Chapter 6 Extra Questions and Answers Human Resource

Human Resource Class 8 Extra Questions and Answer Geography Chapter 6 Very Short Answers Type

Question 1.
What does the population pyramid tells about dependents of a country?
Answer:
Population pyramid tells us how many dependents are there in a country. There are two dependents – young and elderly dependents.

Question 2.
What could be an interesting way of studying the population composition of a country?
Answer:
An interesting way of studying the population composition of a country is by looking at the population pyramid, also called an age-sex pyramid.

Question 3.
How crowded a country is, has little to do with its level of economic development. Give example to show its validity?
Answer:
For example, both Bangladesh and Japan are very densely populated but Japan is far more economically developed than Bangladesh. This is the perfect example for the above’s validity.

Question 4.
Define immigration.
Answer:
When a person enters a new country it is termed as immigration.

Question 5.
What does Emigrations mean?
Answer:
Emigration means when a person leaves a country.

Question 6.
What is the general trend of international migration?
Answer:
The general trend of international migration is from the less developed nations to the more developed nations in search of better employment opportunities.

Question 7.
Who are the Emigrants?
Answer:
The emigrants are the people who leave a country.

Question 8.
What is the meaning of population Density?
Answer:
Population density is the number of people living in a unit area of the earth’s surface.

Question 9.
Which is the ultimate resource?
Answer:
Human Resource.

Question 10.
Which of the two continents are there in which almost 3 quarters of world’s people live?
Answer:
Asia and Africa.

Question 11.
What does industrial area provide?
Answer:
It provides employment opportunities.

Question 12.
What is the most preferrable topography for humans?
Answer:
Plains

Question 13.
Which area of the world has sparse population when it comes to water availability?
Answer:
Deserts.

Question 14.
What led the people to settle in South Africa.
Answer:
Diamond mines of South Africa.

Question 15.
In which year the world’s population reached 3 billion?
Answer:
In 1959.

Question 16.
What was the world’s population in 1804?
Answer:
The world’s population reached one billion.

Question 17.
Why is the population growth slow in the countries like United Kingdom?
Answer:
Because of both low death and low birth rates.

Human Resource Class 8 Extra Questions and Answer Geography Chapter 6 Short Answers Type

Question 1.
What is meant by the term population density and what is the average density of population in whole world and in India?
Answer:
Population density is the number of people living in a unit area of the earth’s surface. It is normally expressed as per sq km. The average density of population in the whole world is 51 persons per sq km. and the average density of population in India is 382 persons sq km. South Central Asia has the highest density of population.

Question 2.
How can we say that people’s are the nation’s greatest resources and human resource is the ultimate resource?
Answer:
People are the nation’s greatest resources as nature’s bounty becomes significant only when people find it useful. It is people with their demands and abilities that turn, them into resources. Hence, human resource is the ultimate resource. Healthy, educated and motivated people develop resources as per their requirements.

Question 3.
What do you know about Human Resource Development (HRD) ministry under the government of India?
Answer:
The Government of India has a ministry of Human Resource Development. The Ministry was created in 1985 with an aim to develop people’s skills. This just shows how important people are as a resource for the country.

Question 4.
What do you mean by the pattern of population distribution? How is world population distributed?
Answer:
The way in which people are spread across the earth’s surface is known as the pattern of population distribution. More than 90% of the world’s population lives in about 30 per cent of the land’s surface. The distribution of world population is extremely uneven.

Question 5.
Define the following:

  1. Birthrate
  2. Death rate
  3. Migrations
  4. Life expectancy

Answer:

  1. Birth Rate: Births are usually measured using the birth rate, i.e. the number of live births per 1,000 people.
  2. Death Rate: Deaths are usually measured using the death rate, i.e. the number of deaths per 1,000 people.
  3. Migrations: It is the movement of people in and out of an area.
  4. Life expectancy: It is the number of years that an average person can expect to live.

Human Resource Class 8 Extra Questions and Answer Geography Chapter 6 Long Answers Type

Question 1.
Define population change. How do the following factors affect the distribution of population?
(i) Topography
(ii) Climate
(iii) Soil
(iv) Water
(v) Minerals.
Answer:
The population change refers to change in the number of people during a specific time. The world population has not been stable.
This is how the following factors affect the distribution of population.

  1. Topography: People always prefer to live on plains rather than mountains and plateaus because these areas are suitable for farming, manufacturing and service activities.
  2. Climate: People usually avoid extreme climates that are very hot or very cold.
  3. Soil: Fertile soils provide suitable land for agriculture. Fertile plains are densely populated around different parts of the world.
  4. Water: People prefer to live in the areas where freshwater is easily available. The river valleys of the world are densely populated while deserts have spare population.
  5. Minerals: Areas with mineral deposits are more populated.

Picture Based Questions Class 8 Geography Chapter 6 Human Resource

Look at the picture given below and answer the following questions:

Question 1.
Class 8 Geography Chapter 6 Extra Questions and Answers Human Resource img-1
(i) When does the population increase?
(ii) When does the condition of steady population arise?
(iii) When does the population decrease?
Answer:
(i) When birth rate is more than death rate.
(ii) When birth rate and death rate both stay the same.
(iii) When death rate is more than birth date.

Map Skills Class 8 Geography Chapter 6 Human Resource

Question 1.
Mark the Annual rates if natural increase in the following colours.
(i) High (2 – 2.9%) – with red colour.
(ii) 0 or decreases – with green colour.
Answer:
Class 8 Geography Chapter 6 Extra Questions and Answers Human Resource img-2

CBSE Sample Papers for Class 12 Political Science All India – 2013

CBSE Sample Papers for Class 12 Political Science All India – 2013

Time Allowed: 3 Hours                                                                                          Maximum Marks: 100
General Instructions:

  1. All questions are compulsory.
  2. Question Numbers 1 to 10 are of one mark each. The answers to these questions should not exceed 20 words each.
  3.  Question Numbers 11 to 20 are of two marks each. The answers to these questions should not exceed 40 words each.
  4. Question Numbers 21 to 30 are of four marks each. The answers to these questions should not exceed 100 words each.
  5. Question Numbers 31 to 35 are of six marks each. The answers to these questions should not exceed 150 words each.
  6. Question Number 35 is based on the map. Write the answer in your Answer-Book.

SET -I

Question.1. What is the main reason for beginning of the US hegemony in 1991 ? 1
Question.2. Define the ‘Bandwagon strategy’. 1
Question.3. Correct the following sentence and rewrite in your answer book:
The International Monetary Fund has five member countries and they enjoy equal status. 1
Question.4. Mention any two new threats to the security of a state. 1
Question.5. Who was popularly known as ‘the milkman of India’ ? 1
Question.6. What was ‘Bombay Plan’ ? 1
Question.7. Name the two main corttenders who contested the election for the post of President of India in 1969.1
Question.8. Who led the Railway strike in 1974 ? What was its main demand ? 1
Question.9. Which organisation of Assam led the movement against foreign nationals in 1979 ? 1
Question.10. Shetkari Sanghatana and Rayata Sangha belonged to which two respective states ?1
Question.11. Which were the two important features of the Soviet System ? 2×1 = 2
Question.12. State any two consequences of the ‘largest garage sale’ in history. 2×1 = 2
Question.13. What is meant by ‘hegemony’ ? 2
Question.14. Name any four main organs of the United Nations. 2
Question.15. What is meant by the non-traditional notion of security ? 2
Question.16. Mention any two merits of Green Revolution. 2×1 = 2
Question.17. Mention any two important features of Bhartiya Jana Sangh’s ideology. 2×1 = 2
Question.18. What were the two main consequences of Indo-Pakistan conflict of 1971 ? 2×1 = 2
Question.19. Highlight any two issues that dominate the politics of North-East India. 2×1 = 2
Question.20. What does a coalition government mean ? Mention any one example of such a government. 1 + 1 = 2
Question.21. Why is the policy of non-alignment of India criticised ? Explain any two reasons. 2×2 = 4
Question.22. Explain any four factors which make the European Union a highly influential organisation. 4×1 = 4
Question.23. Explain the hegemony of United States as a Hard Power. 4
Question.24. Describe India’s relations with China from Independence to 1962. 4
Question.25. State any four criteria that have been proposed in the recent years for new permanent and non-permanent members of the UN Security Council. 4×1 = 4
Question.26. Explain any four factors responsible for the development of globalisation. 4×1 = 4
Question.27. Who founded the Swatantra Party in 1959 ? Describe any three policies and programmes of this party. 1 + 3 = 4
Question.28. Explain India’s Nuclear policy. 4
Question.29. What reasons, do you think, were responsible for the declaration of emergency in 1975 ? Examine any two reasons. 2×2 = 4
Question.30. In the given political outline map of India, four States have been marked (A), (B), (C),and (D). Identify them with the help of the information given below and write their correct names in your answer-book with their respective serial number and the alphabet concerned : 4×1 = 4
(i) The State from where Chipko Movement began.
(ii) The State related to ‘Operation Blue Star’
(iii) The State which was liberated from the control of Portugal in December 1961.
(iv) The State which has been given a special status under Article 370.
cbse-sample-papers-for-class-12-political-science-all-india-2013-1
Question.31. Study the passage given below carefully and answer the questions that follow:
The collapse of Communism was followed in most of these countries by a painful process of transition from an authoritarian socialist system to a democratic capitalist system … Privatisation of state assets and corporate ownership patterns were to be immediately brought in.
Questions: 3×2 = 6
(i) Why has the process of transition been described as painful ?
(ii) Which political system existed before the transition and which system replaced it, if any ?
(iii) What does privatisation imply ?
Or
Study the passage given below and answer the questions that follow :
The two superpowers were keen On expanding their spheres of influence in different parts of the world. In a world sharply divided between the two alliance systems, a state was supposed to remain tied to its protective superpower to limit the influence of the other superpower and its allies…. Most countries of Western Europe sided with the US and those of the Eastern Europe joined the Soviet camp.
Questions:
(i) Name the two superpowers.
(ii) Why did the allies want to remain tied with one of the superpowers ?
(iii) Why did the superpowers want to bring other countries into their camps ?
Question.32. Explain any three environmental concerns in global politics. 3×2 = 6
Or
Explain the concept of globalisation and any two reasons for resistance to it. 2 + 4 = 6
Question.33. “Congress had remained a social and ideological coalition for a long period.” Justify the statement. 6
Or
Assess the outcomes of the early phase of planned development in India.
Question.34. Mention any six steps taken for the restoration of dominance of the Congress Party after the 1971 elections. 6×1 = 6
Or
Mention any six factors responsible for the defeat of the Congress Party in 1977 elections.
Question.35. What are popular movements ? Describe any four benefits of popular movements. 2+4 =6
Or
Highlight any three issues on which a broad agreement has emerged among most of the political parties in India. 3×2 = 6

SET – II

Question.1. What was the objective of founding the European Union ? 1
Question.5. Name any two opposition parties which were part of an electoral alliance in 1971, known as ‘Grand Alliance’. 1
Question.7. Name the present General Secretary of the United Nations who belongs to Asia. 1
Question.9. What was significant about PC Mahalanobis ? 1
Question.10. When was the first democratic election to Sikkim Assembly held and which party swept the elections ? 2
Question.13. Mention the names of any four permanent member states of the UN Security Council. Commissioner ? 1 + 1 = 2
Question.22. Describe any four main events of India’s war with Pakistan in 1965. 4 x 1=4
Question.24. Explain the new economic policies of China since 1978. 4
Question.26. Explain any two causes which led to the formation of ASEAN. 2×2 = 4
Question.28. Explain any four steps taken by the Government of India to curb the emission of green house gases. 4x 1 = 4
Question.35. Describe the growth and characteristics of the Farmers’ Movement led by Bharatiya Kisan Union since 1988. 6
Or
Describe the Assam Movement against the outsiders from 1979 to 1985.

SET – III

Question.2. Name the leader who had given up active politics, but was invited to lead the students’ movement in Bihar in 1974. 1
Question.3. Mention the main function of the World Trade Organisation. 1
Question.9. Who is the ex-officio chairperson of the Planning Commission of India ? 1
Question.13. Which are the two forms of cooperative security as per its traditional notion ? 2×1 = 2
Question.17. Name the pro-Soviet and pro-China factions of the Communist Party of India after the split. 2×1 = 2
Question.22. Explain any four causes of India’s war with Pakistan in 1971. 4×1 = 4
Question.24. How is democratisation going on in Nepal since 1990 ? 4
Question.26. What does SAARC stand for ? Highlight its any three main objectives. 1 + 3 = 4
Question.28. What is the meaning of ‘global commons’ ? Explain any two reasons for the dwindling of global commons. 2×2 = 4
Question.35. What is Sardar Sarovar Project ? What are the achievements of this movement ? Why is it being opposed ? 3×2 = 6
Or
Describe any two secessionist movements of North-East India. 2×3 = 6

CBSE Sample Papers for Class 12 Political Science Delhi – 2013

CBSE Sample Papers for Class 12 Political Science Delhi – 2013

Time Allowed: 3 Hours                                                                                          Maximum Marks: 100
General Instructions:

  1. All questions are compulsory.
  2. Question Numbers 1 to 10 are of one mark each. The answers to these questions should not exceed 20 words each.
  3.  Question Numbers 11 to 20 are of two marks each. The answers to these questions should not exceed 40 words each.
  4. Question Numbers 21 to 30 are of four marks each. The answers to these questions should not exceed 100 words each.
  5. Question Numbers 31 to 35 are of six marks each. The answers to these questions should not exceed 150 words each.
  6. Question Number 35 is based on the map. Write the answer in your Answer-Book.

SET -I

Question.1.What is meant by ‘9/11’ in the context of USA ? 1
Question.2.Correct the following statement and rewrite :
ASEAN stands for ‘Association of South East African Nations’. 1
Question.3.How many Permanent Members and how many Non-permanent Members does the UN Security Council have ? 1
Question.4.What is meant by ‘Security’ ? 1
Question.5.What is the ‘Two Nation Theory’ advanced by the Muslim League ? 1
Question.6.Name the founder President of the Congress Socialist Party. What name was given to this Party after 1948 ? 1
Question.7.After which General Election in India did Congress Party lose its dominance for the first time at the Centre ? 1
Question.8.After the death of Lai Bahadur Shastri, which two leaders of the Congress Party contested against one another to become leader of the Congress Parliamentary Party ? 1
Question.9.Name the popular movement which demanded that no forest exploiting contract should be given to any outsider. 1
Question.10.From which year did the era of Coalition Governments at the Centre begin in India ? 1
Question.11.Name the two superpowers responsible for Cold War. When did the world become unipolar? 1+1=2
Question.12.What is meant by ‘Shock Therapy’ ? 2
Question.13.When was the ASEAN Regional Forum established ? What was its main objective ? 2
Question.14.Explain the traditional concept of ‘Security’. 2
Question.15.Why are International Organizations like the UN required ? 2
Question.16.Which four Princely States of India initially resisted to join-the Indian Union ? 2
Question.17.Which are the two models of development ? Which model of development was adopted by India ? 1+1=2
Question.18.What is India’s Policy of Non-alignment ? 2
Question.19.Mention any two issues of concern related to development projects such as Sardar Sarovar Project. 2×1=2
Question.20. Highlight any two recommendations of the Mandal Commission. 2×1=2
Question.21. Explain any two reasons for the disintegration of the USSR. 2×2=4
Question.22. List any four ‘Directive Principles of State Policy’ given in the Constitution of India for the promotion of international peace and security. 4×1=4
Question.23. Describe any two constraints on American Hegemony. 2×2=4
Question.24. Describe any two major issues of conflict between India and Pakistan leading to the War of 1971. 2×2=4
Question.25. Identify and explain any four new sources of threat to security. 4×1=4
Question.26. Explain any four consequences of globalization. 4×1=4
Question.27. Explain any four reasons for the dominance of the Congress Party in the first three General Elections. 4×1=4
Question.28. Where and when was the organization ‘Dalit Panthers’ formed ? Describe any three of its activities. 1+3=4
Question.29.Why is the ‘Emergency’ and the period around it known as the period of Constitutional crisis ? Explain. 4
Question.30.In the given Political Outline Map of Europe, four member countries of the European Union have been marked A, B, C and D. Identify them with the help of information given below and write their correct names in your answer-book along with their respective serial number and the alphabet concerned 4
(i) An older member of the European Union located between Portugal and France.
(ii) An older member of the European Union located near Belgium and Netherlands.
(iii) Two new members of the European Union.
cbse-sample-papers-for-class-12-political-science-delhi-2013-1
Question.31. Study the passage given below carefully and answer the questions that follow:
The best way to respond to regional aspirations is through democratic negotiations rather than through suppression. Look at the situation in the eighties-militancy had erupted in Punjab; problems were persisting in the North-East; students in Assam were agitating; Kashmir Valley was on the boil. Instead of treating these as simple law and order problems, the Government of India reached negotiated settlement with regional movements.
Questions:
(i) How are regional aspirations dangerous for the unity of the country ?
(ii) What is meant by democratic negotiations ?
(iii) Who was leading the agitation in Assam ?
(iv) What steps were taken by the Government of India to respond to the regional aspirations? 6
Or
Study the passage given below and answer the questions that follow:
In fact, the BSP under Kanshi Ram’s leadership was envisaged as an organization based on pragmatic politics. It derived confidence from the fact that the Bahujans (SCs, STs, OBCs and religious minorities) constituted the majority of the population and were a formidable political force on the strength of their numbers.
Questions:
(i) Which organization was formed by Kanshi Ram ?
(ii) Give the full form of BSP.
(iii) Name any two religious minorities.
(iv) Why are the Bahujans considered a formidable political force ?
Question.32. ‘States have common but differentiated responsibilities towards environment.’ Analyse the statement giving suitable examples. 6
Or
Analyse India’s stand on environmental issues.
Question.33. Explain any three consequences of partition of British India in 1947. 3×2=6
Or
Explain the process and basis of the reorganisation of States of Indian Union. 6
Question.34. ‘Was the declaration of emergency in 1975 necessary ?’ Support your answer with any three suitable arguments. 3×2=6
Or
Explain any three lessons learnt from the emergency imposed in 1975.
Question.35. What was the Soviet System ? Assess any four features of the Soviet System. 2+4=6
Or
Examine the relevance of the Non-aligned Movement after the Cold War era.

SET-II

Question.2. Name the political party of India which was founded by Acharya Narendra Dev before independence. 1
Question.6. What is meant by ‘Congress Syndicate’ ? 1
Question.8. Mention any one objective of the United Nations. 1
Question.10. The States of which region of India are referred to as the ‘Seven Sisters’ ? 1
Question.13. What is meant by ‘Operation Enduring Freedom’ ? 2
Question.17. Highlight the importance of popular movements. 2
Question.22. Explain any four problems before the Election Commission of India for holding the First General Election. 4×1=4
Question.24. Describe the Nuclear Policy of India. 4
Question.26. Describe any four new econorinic policies of China to make it grow at a faster rate. 4×1=4 Question.28. How has technology contributed to globalization ? Explain. 4
Question.34. Examine any six consequences of disintegration of the Soviet Union. 6×1=6
Or
Assess any three reforms proposed by the United Nations Conference on Trade and Development (UNCTAD) as new trade policy for development. 3×2=6

SET -III

Question.2. When was the Constitution of India ready and signed by members of the Constituent Assembly ? When did come into effect ? V2+Vi=l
Question.6. What is meant by Privy Purse ? 1
Question.8. Mention any one function of the World Bank. 1
Question.10. Mention any one point of agreement included in the Rajiv Gandhi-Longowal Accord. 1
Question.13. Before 1971, which two reasons were a source of resentment among the people of East Pakistan (now Bangladesh) against West Pakistan ? 2×1=2
Question.17. Mention any two demands of the Bharatiya Kisan Union. 2×1=2
Question.22. Explain any two differences between the First Five Year Plan and the Second Five Year Plan. 2+2=4
Question.24. Describe the contribution of Jawaharlal Nehru to shape properly the Foreign Policy of India. 4
Question.25. Identify any four new sources of threat to security and explain them. 4×1=4
Question.26. Highlight any four features of’ASEAN’. 4×1=4
Question.28. Explain .any four reasons due t<? which globalization is resisted. 4×1=4
Question.32. Why did the superpowers have military alliances with smaller countries ? Explain any three reasons. 3×2=6
Or
Give any three examples to show that most of the former Soviet Republics were prone to conflicts and tensions.

CBSE Previous Year Solved Papers Class 12 Maths Outside Delhi 2015

CBSE Previous Year Solved  Papers Class 12 Maths Outside Delhi 2015

Time allowed: 3 hours                                                                                          Maximum Marks : 100
General Instructions:

  1. All questions are compulsory.
  2. Please check that this question paper contains 26 questions.
  3. Questions 1-6 in Section A are very short-answer type questions carrying 1 mark each.
  4. Questions 7-19 in Section B are long-answer I type questions carrying 4 marks each.
  5. Questions 20-26 in Section C are long-answer II type questions carrying 6 marks each.
  6. Please write down the serial number of the question before attempting it.

SET I

SECTION – A

Question.1. Write the value of ∆ =
cbse-previous-year-solved-papers-class-12-maths-outside-delhi-2015-1
Solution.
cbse-previous-year-solved-papers-class-12-maths-outside-delhi-2015-2

Question.2. Write the sum of the order and degree of the following differential equation:
cbse-previous-year-solved-papers-class-12-maths-outside-delhi-2015-3
Solution.
cbse-previous-year-solved-papers-class-12-maths-outside-delhi-2015-4

Question.3. Write the integrating factor of the following differential equation:
cbse-previous-year-solved-papers-class-12-maths-outside-delhi-2015-5
Solution.
cbse-previous-year-solved-papers-class-12-maths-outside-delhi-2015-6

Question.4.
cbse-previous-year-solved-papers-class-12-maths-outside-delhi-2015-7
Solution.
cbse-previous-year-solved-papers-class-12-maths-outside-delhi-2015-8

Question.5. Write a unit vector perpendicular to both the vectors
cbse-previous-year-solved-papers-class-12-maths-outside-delhi-2015-9
Solution.
cbse-previous-year-solved-papers-class-12-maths-outside-delhi-2015-10

Question.6. The equations of a line are 5x – 3 = 15y + 7 = 3- 10z. Write the direction cosines of the line.
Solution.
cbse-previous-year-solved-papers-class-12-maths-outside-delhi-2015-11

SECTION – B

Question.7. To promote the making of toilets for women, an organization tried to generate awareness through (i) house calls (ii) letters, and (iii) announcements. The cost for each mode per attempt is given below: (i)Rs 50 (ii) Rs 20 (iii)Rs 40
The number of attempts made in three villages X, Y and Z are given below:
(i)    (ii)      (iii)
400  300    100
300  250      75
500  400       150
Find the total cost incurred by the organization for the three villages separately, using matrices.
Write one value generated by the organization in the society.
Solution. The number of attempts made in three villages X, Y and Z can be represented by the 3 x 3 matrix.
cbse-previous-year-solved-papers-class-12-maths-outside-delhi-2015-12
Hence the total cost incurred by the organization for the three villages separately are Rs 30,000, Rs 23,000 and Rs 39,000.
The organization in the society generated the value of cleanliness for the women welfare.

Question.8.
cbse-previous-year-solved-papers-class-12-maths-outside-delhi-2015-13
Solution. Given
cbse-previous-year-solved-papers-class-12-maths-outside-delhi-2015-14
OR
cbse-previous-year-solved-papers-class-12-maths-outside-delhi-2015-15
Solution.
cbse-previous-year-solved-papers-class-12-maths-outside-delhi-2015-16

Question.9. Using properties of determinants, prove the following:
cbse-previous-year-solved-papers-class-12-maths-outside-delhi-2015-17
Solution.
cbse-previous-year-solved-papers-class-12-maths-outside-delhi-2015-18
cbse-previous-year-solved-papers-class-12-maths-outside-delhi-2015-19

Question.10. Find the adjoint of the matrix.
cbse-previous-year-solved-papers-class-12-maths-outside-delhi-2015-20
Solution.
cbse-previous-year-solved-papers-class-12-maths-outside-delhi-2015-21
cbse-previous-year-solved-papers-class-12-maths-outside-delhi-2015-22
cbse-previous-year-solved-papers-class-12-maths-outside-delhi-2015-23

Question.11. Show that the function f(x) = | x-1 | + | x +1|, for all x ϵ R, is not differentiable at the points x=-1 and x = 1.
Solution.
cbse-previous-year-solved-papers-class-12-maths-outside-delhi-2015-24
cbse-previous-year-solved-papers-class-12-maths-outside-delhi-2015-25
cbse-previous-year-solved-papers-class-12-maths-outside-delhi-2015-26

Question.12.
cbse-previous-year-solved-papers-class-12-maths-outside-delhi-2015-27
Solution.
cbse-previous-year-solved-papers-class-12-maths-outside-delhi-2015-28
cbse-previous-year-solved-papers-class-12-maths-outside-delhi-2015-29

Question.13.
cbse-previous-year-solved-papers-class-12-maths-outside-delhi-2015-30
Solution.
cbse-previous-year-solved-papers-class-12-maths-outside-delhi-2015-31

Question.14.
cbse-previous-year-solved-papers-class-12-maths-outside-delhi-2015-32
Solution.
cbse-previous-year-solved-papers-class-12-maths-outside-delhi-2015-33
cbse-previous-year-solved-papers-class-12-maths-outside-delhi-2015-34
cbse-previous-year-solved-papers-class-12-maths-outside-delhi-2015-35
OR
cbse-previous-year-solved-papers-class-12-maths-outside-delhi-2015-36
Solution.
cbse-previous-year-solved-papers-class-12-maths-outside-delhi-2015-37
cbse-previous-year-solved-papers-class-12-maths-outside-delhi-2015-38

Question.15.
cbse-previous-year-solved-papers-class-12-maths-outside-delhi-2015-39
Solution.
cbse-previous-year-solved-papers-class-12-maths-outside-delhi-2015-40
cbse-previous-year-solved-papers-class-12-maths-outside-delhi-2015-41

Question.16.
cbse-previous-year-solved-papers-class-12-maths-outside-delhi-2015-42
Solution.
cbse-previous-year-solved-papers-class-12-maths-outside-delhi-2015-78
cbse-previous-year-solved-papers-class-12-maths-outside-delhi-2015-43

Question.17.
cbse-previous-year-solved-papers-class-12-maths-outside-delhi-2015-44
Solution.
cbse-previous-year-solved-papers-class-12-maths-outside-delhi-2015-45

Question.18. Find the equation of a line passing through the point (1,2, -4) and perpendicular to two lines
cbse-previous-year-solved-papers-class-12-maths-outside-delhi-2015-46
Solution. Let the direction ratios of required line be a, b, c, t since, the line is perpendicular to
cbse-previous-year-solved-papers-class-12-maths-outside-delhi-2015-47
OR
Find the equation of the plane passing through the points (-1, 2, 0), (2, 2, -1) and parallel to the line
cbse-previous-year-solved-papers-class-12-maths-outside-delhi-2015-48
Solution.
cbse-previous-year-solved-papers-class-12-maths-outside-delhi-2015-49
cbse-previous-year-solved-papers-class-12-maths-outside-delhi-2015-50

Question.19. Three cards are drawn successively with replacement from a well shuffled pack of 52 cards. Find the probability distribution of the number of spades. Hence find the mean of the distribution.
Solution. Let X denote the number of spades when three cards are drawn, then, X is a random variable that can take values 0,1,2,3:
Let E be the event when spade card is drawn,
cbse-previous-year-solved-papers-class-12-maths-outside-delhi-2015-51
cbse-previous-year-solved-papers-class-12-maths-outside-delhi-2015-52
OR
For 6 trials of an experiment, let X be a binomial variate which satisfies the relation 9P(X = 4) = P(X =2). Find the probability of success.
Solution. Let p denote the probability of getting success and q be the probability of failure.
cbse-previous-year-solved-papers-class-12-maths-outside-delhi-2015-53

SECTION – C

Question.20. Consider f: R+ -> [- 9, ∞ ] given by f(x) = 5x + 6x – 9. Prove that f is invertible with
cbse-previous-year-solved-papers-class-12-maths-outside-delhi-2015-54
Solution.
cbse-previous-year-solved-papers-class-12-maths-outside-delhi-2015-55
cbse-previous-year-solved-papers-class-12-maths-outside-delhi-2015-56
OR
cbse-previous-year-solved-papers-class-12-maths-outside-delhi-2015-57
Solution.
cbse-previous-year-solved-papers-class-12-maths-outside-delhi-2015-58

Question.21. Find the value of p for which the curves x2= 9p(9 – y) and x2 = p(y +1) cut each other at right angles.
cbse-previous-year-solved-papers-class-12-maths-outside-delhi-2015-59
cbse-previous-year-solved-papers-class-12-maths-outside-delhi-2015-60

Question.22. Using integration, prove that the curves y2 = 4x and x2 = 4y divide the area of the square bounded by x = 0, x = 4, y = 0 and y = 4 into three equal parts.
Solution.
cbse-previous-year-solved-papers-class-12-maths-outside-delhi-2015-61
cbse-previous-year-solved-papers-class-12-maths-outside-delhi-2015-62
cbse-previous-year-solved-papers-class-12-maths-outside-delhi-2015-63

Question.23.
cbse-previous-year-solved-papers-class-12-maths-outside-delhi-2015-64
Solution.
cbse-previous-year-solved-papers-class-12-maths-outside-delhi-2015-65
cbse-previous-year-solved-papers-class-12-maths-outside-delhi-2015-66
OR
Find the particular solution of the differential equation (tan-1y -x) dy = ( 1 + y2) dx, given that x = 1, when y = 0.
Solution.
cbse-previous-year-solved-papers-class-12-maths-outside-delhi-2015-67
cbse-previous-year-solved-papers-class-12-maths-outside-delhi-2015-68
cbse-previous-year-solved-papers-class-12-maths-outside-delhi-2015-69
cbse-previous-year-solved-papers-class-12-maths-outside-delhi-2015-70
cbse-previous-year-solved-papers-class-12-maths-outside-delhi-2015-71

Question.24. Find the distance of the point P(3,4, 4) from the point, where the line joining the points A(3, – 4, – 5) and B(2, -3,1) intersects the plane 2x + y + z = 7.
Solution. Equation of the line joining the points A(3, – 4, – 5) and B(2, – 3,1) is
cbse-previous-year-solved-papers-class-12-maths-outside-delhi-2015-72

Question.25. A company manufactures three kinds of calculators A, B and C in its two factories I and II, The company has got an order for manufacturing at least 6400 calculators of Kind A, 4000 of kind B and 4800 of kind C. The daily output of factory I is of 50 calculators of kind A, 50 calculators of kind B, and 30 calculators of kind C. The daily output of factory II is of 40 calculators of kind A, 20 of kind B and 40 of kind C. The cost per day to run factory I is Rs 12,000 and of factory II is Rs 15,000. How many days do the two factories have to be in operation to produce the order with the minimum cost ? Formulate this problem as an LPP and solve it graphically.
Solution.
cbse-previous-year-solved-papers-class-12-maths-outside-delhi-2015-73
cbse-previous-year-solved-papers-class-12-maths-outside-delhi-2015-74
cbse-previous-year-solved-papers-class-12-maths-outside-delhi-2015-75

Question.26. In a factory which manufactures bolts, machines A, B and C manufacture respectively 30% 50% and 20% of the bolts. Of their outputs 3,4 and 1 percent respectively are defective bolts. A bolt is drawn at random from the product and is found to be defective. Find the probability that this is not manufactured by machine B.
Solution.
cbse-previous-year-solved-papers-class-12-maths-outside-delhi-2015-76
cbse-previous-year-solved-papers-class-12-maths-outside-delhi-2015-77

CBSE Solved Papers For Class 12 Computer Science (C++) Paper 1

CBSE Solved Papers For Class 12 Computer Science (C++) Paper 1

SECTION A

Question 1.
(a) Explain conditional operator with suitable example ? [2]
(b) Which C++ header file(s) are essentially required to be included to run/execute the following C++code:

void main()
 {
 char *wordl="Hello",*word2="Friends";
 strcat(wordl,word2);
 cout<<word1;
 }

(c) Rewrite the following program after removing the syntactical errors (if. any). Underline each correction.

#include
 #include
 #include
 #include
 class product
 {
 int product_code,qty,price;
 char name[20];
 public:
 product()
 {
 product_code=0 ;qty=0 ,price=0 ;
 name=NULL;
 }
 void entry()
 {
 cout<<"\n Enter code,qty,price"; cin>>product_code>>qty>>price;
 gets(name);
 }
 void tot_price() {return qty*price;}
 } ;
 void main()
 {
 p product;
 p.entry(); cout< }

(d) Write the output of the following C++ program code:
Note: Assume all required header files are already being included in the program.

void change(int *s)
 {
 for(int i=0;i<4;i++)
 {
 if (*s<40)
 {
 if(*s%2==0)
 *s=*s+10;
 else
 *s=*s+11;
 }
 else
 {
 if (*s%2 = = 0)
 *s=*s-10;
 else
 *S=*S-11;
 }
 Cout<<*S<<" ";
 S + + ;
 }
 }
 void main()
 {
 int score[]={25, 60, 35, 53};
 change(score);
 }

(e) Write the output of the following C++ program code :
Note : Assume all required header files are already being included in the program. [3]

class seminar
 {
 char topic[30];
 int charges;
 public:
 seminar()
 {
 strcpy(topic,"Registration");
 charges=5000;
 }
 seminar (char t [] )
 {
 strcpy(topic,t);
 charges=5000
 }
 seminar(int c)
 {
 strcpy(topic,"Registration with Discount");
 charges=5000-c;
 }
 void regis(char t[],int c)
 {
 strcpy(topic,t),
 charges=charges+c;
 }
 void regis(int c=2000)
 {
 charges=charges+c;
 }
 void subject(char t[],int c)
 {
 strcpy(topic,t);
 charges=charges+c;
 }
 void show()
 {
 cout<<topic<<"@"<<charges<<endl;
 }
 } ;
 void main()
 {
 seminar s1, s2(1000), s3("Genetic Mutation"), s4;
 s1.show();
 s2.show();
 si.subject("ICT",2000);
 s1.show();
 s2 .regis ("Cyber Crime" , 2500) ;
 s2.show(); ’
 s3.regis();
 s3.show();
 s4=s2;
 s4.show();
 getch();
 }

(f) Observe the following program carefully and attempt the given questions :

#include
 #include
 #include void main()
 {
 clrscr();
 randomize();
 char courses [] [10]={"M.Tech","MCA","MBA","B.Tech"};
 int ch;
 for(int i=1; i<=3; i++)
 {
 ch=random(i)+ 1;
 cout< }
 getch();
 }

I. Out of all the four courses stored in the variable courses, which course will never be displayed in the output and which course will always be displayed at first in the output ?
II. Mention the minimum and the maximum value assigned to the variable ch ?

Answer:
(a) Conditional operator is also known as ternary operator because it requires three operands and can be used to replace simple if-else code. It is used to check the condition and execute first expression if condition is true else execute other.
Syntax:
Conditional expression? Expression 1 : Expression 2;
Explanation:
If the conditional expression is true then expression 1 executes otherwise expression 2 executes.
Example:
int y=10,x;
x=y>10?l:0;
cout<<x;
Output: 0

(b) iostream.h
string.h

(c) #include
 #include
 #include
 #include
 class product
 {
 int product_code,qty,price;
 char name[20];
 public:
 product(){
 product_code=0;qty=0;price=0;
 strcpy(name,NULL);
 void entry()
 {
 cout<<"\n Enter code,qty,price"; cin> >product_code > >qty>>price;
 gets(name);
 }
 int tot_price() {return qty*price;}
 };
 void main()
 {
 product p;
 p.entry();
 cout< }

(d) 36 50 46 42
(e) Registration@5000
Registration with Discount@4000
ICT@7000
Cyber Crime@6500
Genetic Mutation@7000
Cyber Crime@6500
(f) I. M.Tech will never be displayed in the output.
MCA will always be displayed at first in the output.
II. Minimum value of ch=1
Maximum value of ch=3

Question 2.
(a) What do you understand by Function overloading or Functional polymorphism? Explain with suitable example.
(b) Answer the questions(i) and (ii) after going through the following class :

class planet
 {
 char name[20];char distance[20];
 public:
 planet() //Function 1
 {
 strcpy(name, "Venus");
 strcpy(distance,"38 million km");
 }
 void display(char na[],char d[]) //Function 2
 {
 cout<<na<<"has "<<d<<" distance from Earth"<<endl;
 }
 planet (char na[], char d[]) //Function 3
 {
 strcpy(name,na);
 strcpy(distance,d);
 }
 -planet() //Function 4
 {
 cout<<"Planetarium time over!!!"<<endl;
 }
 };

I. What is Function 1 referred as? When will it be executed ?
II. Write suitable C++ statement to invoke Function 2.
(c) Define a class DanceAcademy in C++ with following description :
Private Members

  • Enrollno of type int
  • Name of type string
  • Style of type string
  • Fee of type float
  • A member function chkfee() to assign the value of fee variable according to the style entered by the user according to the criteria as given below :
Style Fee
Classical 10000
Western 8000
Freestyle 11000

Public Members

  • A function enrollment () to allow users to enter values for Enrollno, Name, Style and call function chkfee()to assign value of fee variable according to the Style entered by the user.
  • A function display () to allow users to view the details of all the data members.

(d) Answer the questions (i) to (iv) based on the following :

class indoor_sports
 {
 int i_id;
 char i_name[20] ;
 char i_coach [2 0]
 protected:
 int i_rank,i_fee;
 void get_ifee();
 public:
 indoor_sports();
 void iEntry();
 void ishow();
 } ;
 class outdoor_sports
 {
 int o_id;
 char o_name[20];
 char o_coach[20]; protected:
 int orank,ofee;
 void get_ofee();
 public:
 outdoor_sports();
 void oEntry();
 void oshow();
 } ;
 class sports:public indoor_sports,protected outdoor_sports
 {
 char rules [20];
 public:
 sports ();
 void registration();
 void showdata();
 } ;

(i) Name the type of inheritance illustrated in the above C++ code.
(ii) Write the names of all the members, which are accessible from the objects belonging to class outdoor_sports.
(iii) Write the names of all the member functions, which are accessible from the member function of class sports.
(iv) What will be the size of the object belonging to class indoor_sports ?

Answer:
(a) It is a method of using the same function or method to work using different sets of input. Function overloading is one of the example of polymorphism, where more than one function carrying same name behave differently with different set of parameters passed to them,

void show()
 {
 cout<<"\n Hello World!";
 }
 void show(char na[])
 {
 cout<<"\n Hello World! Its "<<na;
 }

(b) I. Constructor
It will be executed at the time of object creation.
II. Planet P; p.display (“Pluto”, 7.5 Billion km”);
It will be executed at the time of object creation.

(c) class DanceAcademy
 {
 int Enrollno;
 char Name[20];
 char Style[20];
 float Fee ;
 void chkfee()
 {
 if(strcmpi(Style, "Classical")==0)
 Fee=10000;
 else if(strcmpi(Style, "Western")==0)
 Fee=8000;
 else if(strcmpi(Style, "Freestyle")==0)
 Fee=11000;
 }
 public:
 void enrollment()
 {
 cout<<"Please enter Enrollno,Name,Style"; cin>>Enrollno;
 gets(Name);
 gets(Style);
 chkfee();
 }
 void display()
 {
 cout<<"\n Entered Enrollno, Name, Style and Fee is:
 "<<Enrollno<<"\t"<<Name<<"\t"<<Style<<"\t"<<Fee;
 }
 } ;

(d) (i) Multiple Inheritance
(ii) Data Members: None
Member Functions: oEntry(), oShow()
(iii) registration() , showdata(), oEntry(), oShow(), get_ofee(), iEntry() , iShow(), get_ifee()
(iv) 46 Bytes

Question 3.
(a) Write the definition of a function grace_score (int score [], int size) in C + + , which should check all the elements of the array and give an increase of 5 to those scores which are less than 40.
Example : if an array of seven integers is as follows :
45, 35, 85, 80, 33, 27, 90
After executing the function, the array content should be changed as follows :
45, 40, 85, 80, 38, 32, 90
(b) An array P[30][20] is stored along the column in the memory with each element requiring 2 bytes of storage. If the base address of the array P is 26500, find out the location of P[20][10].
(c) Write the definition of a member function push() for a class Library in C++ to insert a book information in a dynamically allocated stack of books considering the following code is already written as a part of the program :

struct book
 {
 int bookid;
 char bookname[20];
 book *next;
 };
 class Library
 {
 book *top;
 public:
 Library()
 {
 top=NULL;
 }
 void push();
 void pop();
 void disp();
 -Library () ;
 };

(d) Write a user-defined function swap_row(int ARR[ ][3], int R, int C) in C++ to swap the first row
values with the last row values :
For example if the content of the array is :

10 20 30
40 50 60
70 80 90

Then after function call, the content of the array should be :

70 80 90
40 50 60
10 20 30

(e) Evaluate the following POSTFIX expression. Show the status of Stack after execution of each operation separately:
45, 45, +, 32, 20, 10, /, -, *

Answer:

(a) void grace_score (int score [],int size)
 {
 for(int i=0; i<size; i++)
 {
 if(score[i]<40)
 score[i]=score[i]+5;
 cout<<score[i]<<" " ;
 }
 }

(b) Total number of rows= 30
Total size= 2 bytes
Base Address = 26500
LOC (P[I][J]) = BaseAddress+((I-LBR) + (J-LBC) * R)*W
Assuming Lower Bound of Row(LBR)=0
Lower Bound of Column(LBC)=0
Total number of Rows(R)=30
Size of each element(W)=2
LOC(P[20][10])= 26500 +((20-0)+(10-0)*30)*2
LOC(P[20][10])= 26500 +640
LOC(P[20][10])= 27140

(c) void Library::push()
 {
 book *nptr;
 nptr=new book;
 cout<<"Enter values for bookid and bookname"; cin>>nptr->bookid;
 gets(nptr->bookname);
 nptr->next=NULL;
 if(top==NULL)
 top=nptr;
 else
 {
 nptr->next=top;
 top=nptr;
 }
 }
(d) void swap_row(int ARR[][3],int R,int C)
 {
 for(int i=0,j=0;j<C;j++)
 {
 int temp=ARR[i][j];
 ARR[i] [j] =ARR[R-1] [j] ;
 ARR[R-1][j]=temp;
 }
 }

(e)

Element Scanned Stack Status
45 45
45 45, 45
+ 90
32 90, 32
20 90, 32, 20
10 90, 32, 20, 10
/ 90, 32, 2
90, 30
* 2700

Hence the final result is 2700

Question 4.
(a) Find the output of the following C+ + code considering that the binary file sp.dat already exists on the hard disk with 2 records in it.

class sports
 {
 int id;
 char sname[20];
 char coach[20];
 public:
 void entry();
 void show();
 void writing();
 void reading();
 }s;
 void sportsreading()
 {
 ifstream i;
 i.open("sp.dat");
 while(1)
 {
 i.read((char*)&s,sizeof(s) ) ;
 if (i. eof())
 break;
 else
 cout<<"\n"<<i.tellg(); 
} 
i.close(); 
} 
void main() 
{ 
s.reading(); 
}

(b) Write a user defined function word_count() in C+ + to count how many words are present in a text file named “opinion.txt”. For example, if the file opinion.txt contains following text:
Co-education system is necessary for a balanced society. With co-education system, Girls and Boys may develop a feeling of mutual respect towards each other.
The function should display the following :
Total number of words present in the text file are : 24
(c) Write a function display () in C++ to display all the students who have got a distinction (scored percentage more than or equal to 75) from a binary file “stud.dat”, assuming the binary file is containing the objects of the following class :

class student 
{ 
int rno;
 char sname [20]; 
int percent; 
public: 
int retpercent() 
{ 
return percent; 
} 
void getdetails() 
{ 
cin>>rno;
 gets(sname);
 cin>>percent;
 }
 void showdetails()
 {
 cout<<rno;
 puts(sname);
 cout<<percent;
 }

Answer:
(a) 42
84

(b) void word_count()
 {
 ifstream i;char ch[20];int c=0;
 i.open("opinion.txt ");
 while (! i .eof ())
 {
 i>>ch;
 C=C+1;
 }
 cout<<" Total number of words present in the text file are: "<<c; }
(c) void display() 
{ 
student s; 
ifstream i("stud.dat"); 
while(i.read((char*)&s,sizeof(s))) 
{ 
if(s.retpercent()>=75)
 s.showdetails ();
 }
 i.close();
 }

SECTION B

Question 5.
(a) Observe the table ‘Club’ given below :

Member_id Member_Name Address Age Fee
M001 Sumit New Delhi 20 2000
M002 Nisha Gurgaon 19 3500
M003 Niharika New Delhi 21 2100
M004 Sachin Faridabad 18 3500

(i) What is the cardinality and degree of the above given table ?
(ii) If a new column contact_no has been added and three more members havg joined the club then how these changes will affect the degree and cardinality of the above given table.
(b) Write SQL commands for the queries (i) to (iv) and output for (v) to (viii) based on the tables ‘Watches’ and ‘Sale’ given below.
Watches

Watchid Watch_Name Price Type Qty_Store
woo1 HighTime 10000 Unisex 100
W002 LifeTime 15000 Ladies 150
W003 Wave 20000 Gents 200
W004 HighFashion 7000 Unisex 250
W005 GoldenTime 25000 Gents 100

Sale

Watchid Qty_Sold Quarter
W001 10 1
W003 5 1
W002 20 2
W003 10 2
W001 15 3
W002 20 3
W005 10 3
W003 5 4

(i) To display all the details of those watches whose name ends with ‘Time’
(ii) To display watch’s name and price of those watches which have price range in between 5000 -15000.
(iii) To display total quantity in store of Unisex type watches.
(iv) To display watch name and their quantity sold in first quarter.
(v) select max(price), min(qty_store) from watches;
(vi) select quarter, sum(qty_sold) from sale group by quarter;
(vii) select watchname,price,type from watches w, sale s where w.watchid!=s.watchid; (viii)select watch_name, qty_store, sum(qty_sold), qty_storesum(qty_sold) “Stock” from watches w, sales where w.watchid=s.watchid group by s.watchid;

Answer:
(a) (i) Cadinality: 4
Degree: 5
(ii) Cardinality: 7
Degree: 6
(b) (i) select * from watches where watchname like ‘%Time’
(ii) select watch name, price from watches where price between 5000 and 15000;
(iii) select sum(qtystore) from watches where type like ‘Unisex’;
(iv) select watch_name,qty_sold from watches w,sale s where w.watchid=s.watchid and quarter=1;
(v)

max(price) min(qty_store)
25000 100

(vi)

Quarter sum(qtysold)
1 15
2 30
3 45
4 15

(vii)

watch_name price type
High Fashion 7000 Unisex

(viii)

watch_name qty_store qty_sold Stock
High Time 100 25 75
Life Time 150 40 110
Wave 200 30 170
Golden Time 100 10 90

Question 6.
(a) Correct the following boolean statements :
1. X+1= X
2. (A’)’ = A’
3. A+A’ = 0
4. (A+B)’ = A.B
(b) Draw the equivalent logic circuit for the following Boolean expression : (A.B) + C
(c)Write the POS form of a Boolean Function F, which is represented in a truth table as follows:

P Q R F
0 0 0 0
0 0 1 1
0 1 0 1
0 1 1 1
1 0 0 0
1 0 1 1
1 1 0 0
1 1 1 1

(d) Reduce the following Boolean Expression using K Map :
F(A,B,C,D) = S (0, 1, 3, 5, 6, 7, 9, 11, 13, 14, 15)

Answer:
(a) 1. X+l =1 or X+0=X
2. ((A’)’) = A
3. A + A’= 1 or A . A’=0
4. (A+B)’ = A’. B’
(b)
cbse-solved-papers-for-class-12-computer-science-c-paper-1-2
(c) (P+Q+R).(P’+Q+R).(P’+Q’+R)
(d)
cbse-solved-papers-for-class-12-computer-science-c-paper-1-3
A’B’C’ + D + BC

Question 7.
(a) Identify the type of topology on the basis of the following : [2]
(i) Since every node is directly connected to the server, a large amount of cable is needed which increases the installation cost of the network.
(ii) It has a single common data path connecting all the nodes.
(b) Expand the following :
(i) VOIP
(ii) SMTP
(c) Who is a hacker ?
(d) The following is a 32 bit binary number usually represented as 4 decimal values, each representing 8 bits, in the range 0 to 255 (known as octets) separated by decimal points.
140.179.220.200
What is it ? What is its importance ?
(e) Daniel has to share the data among various computers of his two offices branches situated in the same city. Name the network (out of LAN, WAN, PAN and MAN) which is being formed in this process.
(f) Rehaana Medicos Center has set up its new center in Dubai. It has four buildings as shown in the diagram given below:
cbse-solved-papers-for-class-12-computer-science-c-paper-1-1
Distance between various buildings are as followa:

Accounts to Research Lab 55 m
Accounts to Store 150 m
Store to Packaging Unit 160 m
Packaging Unit to Research Lab 60 m
Accounts to Packaging Unit 125 m
Store to Research Lab 180 m

Number of Computers

Accounts 25
Research Lab 100
Store 15
Packaging Unit 60

As a network expert, provide the best possible answer for the following queries :
(i) Suggest a cable layout of connections between the buildings.
(ii) Suggest the most suitable place (i.e. buildings) to house the server of this organization.
(iii) Suggest the placement of the following device with justification :
(a) Repeater
(b) Hub/Switch
(iv) Suggest a system (hardware/software) to prevent unauthorized access to or from the network.

Answer:
(a) (i) Star Topology
(ii) Bus Topology
(b) (i) Voice Over Internet Protocol
(ii) Simple Mail Transfer Protocol
(c) A computer enthusiast, who uses his computer programming skills to intentionally access a computer without authorization is known as hacker. A hacker accesses the computer without the intention of destroying data or maliciously harming the computer.
(d) It is an IP Address.lt is used to identify the computers on a network.
(e) MAN
(f) (i) Layout 1
cbse-solved-papers-for-class-12-computer-science-c-paper-1-4
Layout 2
cbse-solved-papers-for-class-12-computer-science-c-paper-1-5
(ii) The most suitable place/ building to house the server of this organization would be building Research Lab, as this building contains the maximum number of computers.
(iii) (a) For layoutl, since the cabling distance between Accounts to Store is quite large, so a repeater would ideally be needed along their path to avoid loss of signals during the course of data flow in this route. For layout2, since the cabling distance between Store to Research Lab is quite large, so a repeater would ideally be placed.
(b) In both the layouts, a Hub/Switch each would be needed in all the buildings to interconnect the group of cables from the different computers in each building.
(iv) Firewall

CBSE Solved Papers For Class 12 Computer Science (C++) Paper 3

 CBSE Solved Papers For Class 12 Computer Science (C++) Paper 3

SECTION A

Question 1.
(a) Out of the following, find those identifiers, which cannot be used for naming variable, Constants or Functions in a C++ program :

_Cost, Price*Qty, float, Switch,
 Address One, Delete, Numberl2, do

(b) Jayapriya has started learning C++ and has typed the following program. When she complied the following code written by her, she discovered that she needs to include some header files to successfully compile and execute it. Write the names of those header files, which are required to be included in the code.

void main ()
 {
 float A, Number, outcome;
 cin>>A>>Number ;
 outcome=pow (A, Number);
 cout<<outcome<<endl;
 }

(c) Find the output of the following program:

#include
#include
 void main ()
 {
 char Text [] = {"Mind 0 Work
 for (int I = 0; Text [I] ! = '/O', I++)
 {
 if (!isalpha (Text [I]))
 Text [I]='*';
 else if (isupper (Text [I]))
 Text [I]=Text [I] +1;
 else
 Text [I]=Text [1+1];
 puts (text);
 }
 }

(d) Rewrite the following program after removing the syntactical error(s), if any. Underline each correction.

#include<iostream.h>
 const int size=5,
 void main ()
 {
 int Array (size);
 Array={50, 40, 30, 20, 10);
 for int (ctr=0; ctr<size; ctr++); 
cout>>Array [ctr]; }

(e) Find and write the output of the following C+ + program code:
Note: Assume all required header files are already being included in the program.

class market
 {
 long int code;
 float Rate;
 int DD;
 public:
 market () { code = 1000; Rate = 100; DD = 1:}
 void Getcode (long int C, float R)
 {
 code = C;
 Rate = R;
 }
 void Update (int change, int D)
 {
 Rate + = Change;
 DD = D;
 }
 void Status ()
 {
 cout < < "Date : " < < DD < < endl;
 cout < < code < < "#" < < Rate < < endl;
 }
 };
 void main ()
 {
 Market S, T, U;
 S. Getcode (1324, 350);
 T. Getcode (1435, 250);
 S. Update (50, 28);
 U. Update (-25, 26);
 S. Status ();
 T. Status ();
 U. Status ();
 }

(f) Observe the following C+ + code and find out, which out of the given options (i) to (iv) are the expected correct output.Also assign the maximum and minimum value that can be assigned to the variable ‘Go’.

void main()
 { int X [4] ={100, 75, 10, 125};
 int Go = random(2)+2;
 for (inti = Go; i< 4; i++)
 cout<<X[i]<<"$$";
 }
 i. 100$$75 ii. 75$$10$$125$$ iii. 75$$10$$ iv,10$$125$

Answer:

 (a) Price*Qty,
 float,
 Address one,
 do_cost

(b) iostream.h or iomanip.h
math.h
Note:
Ignore any other header files, if mentioned.
complex. h is acceptable in place of match.h
(c) The output of the above program is:
ind ***or k**
(d) The correct program is as follows:

#include
 const int size=5;
 void main ()
 {
 int Array [size];
 Array [ ]={50, 40, 30, 20, 10}
 for (int ctr=0; ctr<size; ctr++)
 cout << Array [ctr];
 }

(e) Date: 28
1324#400
Date : 1
1435#250
Date : 26
1000#75

(f) iv is the correct option.
Minimum value of Go = 2
Maximum value of Go = 3

Question 2.
(a) What is function overloading ? Write an example using C++ to illustrate the concept of function overloading.
(b) Observe the following C++ code and answer the question (i) and (ii):

class E and I
 {
 int Temperature, Humidity;
 char city[30];
 public :
 E_and_I() //Function 1
 {
 Temperature=0; Humidity=0;
 cout<<"set to Zero"<<endl;
 }
 void E and I (int T, int H, char c[]) //Function2
 {
 Temperatrue T; .
 Humidity H;
 strcpy(city, c) ;
 }
 void show() //Function3
 (
 cout«"Temperature«"; "<<Humidity«endl;
 puts(city);
 }
 ~E_and_I //Function4
 ' {
 cout<<"Data Removed! "<<endl;
 }
 }

(i) Fill in the blank lines as Statement 1 and Statement 2 to execute Functions 2 and 3 respectively in the following code:

void main()
 {
 E and I E;
 ............... //Statement 1
 .............. //Statement 2
 } //The end of main function here

(ii) Which function will be executed at the point where “//The end of main()function here” is written in the above code ? What is this function called and executed here is known as ?
(c) Answer the questions (i) to (iv) based on the following code:

class Teacher
 {
 char TNo [50], TName [20], Dept [10];
 int Workload;
 protected:
 float Salary;
 void AssignSal (Float);
 Teacher ();
 void TEntry ();
 void TDisplay ();
 };
 class student
 {
 char Admno [10],
 SName [20],
 Stream [10];
 protected:
 int Attendence, Totmarks;
 public:
 student ();
 void SEntry ();
 void SDisplay () ;
 };
 class School: public student, public Teacher
 {
 char SCode [10], SchName [20];
 public:
 School ();
 void SchEntry ();
 void SchDisplay ();
 };

(i) Which type of inheritance is depicted by the above example?
(ii) Identify the member function(s) that cannot be called directly from the object of the class School from the following:
TEntry ()
SDisplay ()
SchEntry ()
(iii) Write name of all the member(s) accessible from member function of class School.
(iv) If class school was derived privately from class Teacher and privately from class Student, then, name the member function (s) that could be accessed through object of class school.
(d) Define a class TEACHER with the following specifications:
Private Members:
name: 20 characters
subject 10 characters
basic, da, hra float
Salary float
calculate () A function that computes the salary and returns it. Salary is sum of basic, da and hra.
Public Members:
readata () A function that accept data values and invokes the calculate function,
displaydata () A function that prints the data on the screen.

Answer:
(a) C++ enables us to create more than one function with the same name. This is called function overloading. The functions must differ in their parameter list, with a different type of parameter, different number of parameters, or both.

int double(intA)
 {
 return(2*A);
 long Double(long A)
 {
 return(2*A);
 }
 float Double (float A)
 {
 return (2*A);
 }

(b) (i) Statement 1: E.E_and_I(32,5, “Indore”);
Statement 2 : E. Show ();
(ii) Function 4 will be executed. This function is called as destructor, which is excuted when object goes out of scope. [1]
(c) (i) Multiple Inheritence is depicted by the given example. [1]
(ii) TEntry (), as it is declared protected, write, other two are public.

(iii) Teacher ()
 Tentry ()
 TDisplay ()
 Student ()
 SEntry ()
 SDisplay ()
 School ()
 SchEntry ()
 SchDisplay ()
 Salary ()
 Assign Sal ()
 Attendence
 Totmarks
 SCode
 SchName

(iv) The member functions are:

SchEntry ()
 SchDisplay ()

(d) class TEACHER
 {
 char name [20];
 char subject [10];
 float basic, da, hra;
 float salary;
 float calculate ()
 {
 return (basic + da + hra);
 }
 public:
 void readata ( )
 {
 cout<<"\n Enter the teacher name:";
 gets(name);
 cout<<"\n Enter the Subject;";
 gets(subject);
 cout<<"\n Enter the basic salary:"; cin>>basic;
 cout<<"\n Enter the HRA:"; cin>>hra;
 salary=calculate ();
 }
 void displaydata ()
 {
 cout <<"\n Teacher's name is : ";puts (name) ;
 cout<<"\n Subject taught is:";puts(subject);
 cout<<"\n Basic salary is:"<<basic;
 cout<<"\n Da is"<<da;
 cout<<"\n HRA is;"<<hra;
 cout<<"\n Total salary is:"<<salary;
 }
 } ;

Question 3.
(a) Write a C+ + program to sort an array of 10 integers. Sort array showing status of the array after each pass. Assume 10 integers.
(b) An integer array A [40] [30] is stored along the row in the memory. If the element A[20] [25} is stored at 50000. Find out the location of A [7] [10].
(c) Write a function in C++ to print the sum of all the values which are either divisible by 2 or divisible by 3, present in a two dimensional array passed as the argument to the function.
(d) Write a function in C++ to delete an element from a dynamically allocated queue where each node contains a real number as data. Assume the following definition of MYNODE for the same

struct MYNODE
 {
 float NUM;
 MYNODE * Link;
 };

(e) Evaluate the following postfix notation of expression (show status of stack after execution of each operation);
5, 20, 15, -, *, 25, 2, *, +

Answer:

(a) /* Sorting Array and Showing Status of Array After Each Pass*/
 #include
 #include
 void main ()
 {
 int i, array [10], n, j, temp,small, pos;
 clrscr ();n=10;/given in question
 cout<<"Enter size of an array"; cin>>n;
 for (i=0; i<n;i++)
 {
 //Inserting array elements.
 cout<<"Insert"<>array [i];}
 for (i=0;i<10;i++)//program code to sort array
 {small=array[i];
 pos=i;
 for(j=0;j<g;j++)
 {if(array[j]<small)
 {small=array[j];pos=j;}
 }
 tmp=atray[i];
 array[i]=array[pos];
 array[pos]=tmp;
 cout<<"\nArray other pass-"< for(j=0;<10;j ++)
 cout<<array[j]
 }
 //Displaying sorted array.
 cout<<"\n Sorted Array Element Are";
 for (i=0; i<n; i++)
 cout<< array [i] ;
 getch ();
 }
(b)A[i J [ j ] =B+Wx[No.of columns (I-Lr) + (J-Lc ]
 A [ 2 0 ] [ 25 ]=B+2x[ 30x(20-0)+(25-0)]
 5000=B+2x[3Ox (20-0) + (25-0) ]
 B=48750
 A[7][10]=4875Q+2x[30x(7-0)+(10-0)]
 =49190
(c) void Div_2_or_3 (int A [ ] [ ] , int N, int M)
 {
 int Sum=0;
 for (int i=0; i<N; i++)
 for (int j=0; j<M; j++)
 if (A [i] [ j ] %2= = 0 || A[i] [ j ] %3= =0)
 Sum=Sum+A[i][j];
 cout<<Sum;
 }

(d) void QueDel(MyNode*front)
 {
 MYNODE*temp;
 if (front==NULL)
 cout<<"Queue empty" else { temp=front; front=front—>Link;
 delete temp;
 }
 }

(e) Postfix expression :
5, 20, 15, -, *, 25, 2, *, +

Step Input Operation Stack Status
1 5 PUSH 5 5
2 20 PUSH 20 5   20
3 15 PUSH 15 5 20   15
4 POP 15
POP 20
Calculate:
20-15=5
PUSH 5 5 5
5 * POP 5
POP 5
Calculate:
5*5=25
PUSH 25 25
6  25 PUSH 25 25 25
7 2 Push 2 25 25 2
8 * POP2
POP 25
Calculate:
25*2=50
PUSH 50 25 50
9  + POP 50
POP 25
Calculate:
25 +50 = 75
PUSH 75 75

Ans. 75

Question 4.
(a) Write a definition for function Economic () in C++ to read each record of a binary file ITEMS. DAT, find and display those items, which costs less than 2500. Assume that the file is created with the help of objects of class ITEMS, which is defined below :

class ITEMS
 {
 int ID; char GIFT [20] ;float cost;
 public :
 void Get ()
 {
 cin>>ID;gets (GIFT) ;cin>>cost;
 }
 void See()
 {
 cout<<ID<<": "GIFT<<": "<<Cost<<endl;
 }
 float Get Cost()
 {
 return Cost;
 }
 }

(b) Write function definition for lower() in C++ to read the content of a text file Book.txt, and count all those leters which are in lowercase.
(c) Find the output of the following C++ code considering that the binary file CLIENTS.DAT exists on the hard disk with records of 100 members.

class Clients
 {
 int cno;charName[20];
 public:
 void In();void out();
 void main()
 {
 fstream CF;
 CF.open("CLIENTS.DAT",ios::binary : ios::in);
 CLIENTS C; ,
 CF.read((char*)&c;sizeof (c));
 CF.read ((char*) &c; sizeof (c) ) ;
 CF.read((char*)&C, sizeof (c));
 int P0S=CF.tellg()/sizeof (c);
 cout<<"PRESENT RECORD: "<<P0S<<endl;
 CF.close();
 }

Answer:

(a) void Economic()
 {
 ifstream f;
 f.open("ITEMS.DAT",ios::binary);
 ITEMS i;
 while(f.read((char*)&i, size of (i))
 {
 if(i.GetCost()<2500)
 i.See ();
 }
 f.close ();
 }

(b) Function in C++ to count the number of lowercase letters in a text file “Book. Txt”:

void lower()
 {
 char ch;
 int count =0;
 ifstream infile;
 infile.open ("Book.Txt") ;
 if (! infile)
 {
 cout << "can't open the file";
 exit ( ) ;
 }
 (while infile, get (ch) !=0)
 {
 if (islower(ch))
 {
 count ++;
 }
 }
 cout <<count;
 }

(c) POS=66/22 = 3
Ans:
Present Record: 3

SECTION B

Question 5.
(a) Give some advantages of DBMS.
(b) Consider the following tables consignor and consignee. Write SQL commands for the statements (i) to (iv) and give the outputs for SQL queries (v) to (viii):
Table: Consignor

CnorlD Cnor Name Cnor Address  City
ND 01 R singhal 24, ABC Enclave New Delhi
ND 02 Amit Kumar 133, Palm Avenue New Delhi
MU 15 R Kohli 5/A, South street Mumbai
MU 50 S Kaur 27-K, Westend Mumbai

Table: Consignee

Cnee ID Cnor ID Cnee Name Cnee Address Cnee City
MU 05 ND 01 Rahul Kishore 5, Park Avenue Mumbai
ND 08 ND 02 P Dhingra 16/J, Moore Enclave New Delhi
K 019 MU 15 APRoy 24, Central Avenue Kolkata
MU 32 ND02 S Mittal , P-254, AB Colony Mumbai
ND48 MU 50 BP Jain B, Block D, A vihar New Delhi

(i)To display the name of all consignors from Mumbai
(ii) To display the Cnee ID, Cnor Name, Cnor Address, Cnee Name, Cnee Address for every Consignee
(iii) To display consignee details in ascending order of Cnee Name
(iv) To display number of consignors from each city

(v) SELECT DISTINCT City FROM CONSIGNEE
 (vi) SELECT A. Cnor Name, B. Cnee Name
 FROM CONSIGNOR A, CONSIGNEE B
 WHERE A. Cnor ID=B. Cnor ID AND
 B. Cnee City='Mumbai';
 (vii) SELECT Cnee Name, Cnee Address
 FROM CONSIGNEE
 Where Cnee City Not ( 'Mumbai' , 'Kolkata' ) ;
 (viii) SELECT Cnee ID, Cnee Name
 FROM CONSIGNEE
 WHERE Cnor ID='MU 15' or Cnor ID='ND01'

Answer:
(a) Some of the advantages of DBMS are:
(i) DBMS helps to make the data management more efficiently and effectively.
(ii) DBMS provides end users better access to more and better managed data.
(iii) DBMS provides user security and data privacy within the database.
(iv) DBMS provides back up and recovery procedures to ensure data safety and integrity.

(b) (i) SELECT Cnor Name from CONSIGNOR WHERE CITY="Mumbai";
 (ii) SELECT B. Cneee.ID, A. Cnor Name, A.Cnor Address, B.Cnee Name, B.Cnee Address FROM CONSIGNOR A, CONSIGNEE B WHERE B. Cnor ID=A Cnor ID;
 (iii) SELECT * FROM CONSIGNEE
 ORDER BY Cnee Name;
 (iv) SELECT COUNT (City)
 FROM CONSIGNOR
 GROUP BY CITY;

(v) Mumbai
New Delhi
Kolkata
(vi) R Singhal
Rahul Kishore
Amit Kumar
S. Mittal
(vii) P Dhingra
16/j, Moore Enclave
BP Jain
13, Block D, A Vihar
(viii) K019
APROY
MU 05
Rahul Kishore

Question 6.
(a) Verify the following using Boolean Laws
X + Y’ = X.Y + X.Y’ + X’.Y’
(b) Write the Boolean Expression for the result of the Logic Circuit as shown below :
cbse-solved-papers-for-class-12-computer-science-c-paper-3-1
(c) Write the SOP form of a Boolean function F, which is represented in a truth table as follows:

A B C D
0 0 0 0
0 0 1 1
0 1 0 1
0 1 1 0
1 0 0 1
1 0 1 1
1 1 0 0
1 1 1 0

(d) Reduce the following Boolean expression using K-map.
F (A, B, C, D) = I (0, 1, 2, 4, 5, 8, 9, 10, 11)

Answer:
(a) X + Y’ = X.Y + X.Y’ + X’.Y’
Taking R.H.S.
X.Y + X.Y’ + X’.Y’
= X (Y + Y’) + X’.Y’ (grouping)
= X (1) + X’.Y’ (X + X’ = 1)
= X + Y’ (Absorption law)
Hence Verified.

(b) ( (U+V’ ) . (U+W) ) . (V+W’ )
OR
(U+V’).(U+W).(V+W’)

(c) F=A’B’C+A’BC’+AB’C’+AB’C

(d) F (A,B,C,D) = X (0,1,2,4,5,8,9,10,11)
K-Map is as following:
cbse-solved-papers-for-class-12-computer-science-c-paper-3-2
F (A,B,C,D) = AC + AB + BD

Question 7.
(a) Explain one difference between circuit switching and packet switching.
(b) Write the following abbreviations in their full form:
FTP, WAN, WWW
(c) What is NFS?
(d) Write two advantages and two disadvantages of following topologies in network:
(i) BUS Topology
(ii) STAR Topology

Answer:
(a) Packet switching: Packet switching is a combination of both circuit and message switching and it divides the lengthy message in small parts known as Packets.
Circuit Switching: Circuit Switching establishes a communication path between sending and receiv¬ing computers. The path is connected sequence of links between different Switching Nodes.

(b) FTP stands for File Transfer Protocol WAN stands for Wide Area Network WWW stands for World Wide Web.

(c) NFS: NFS stands for Network File System. It is an operating system which allows all network users to access the shared files stored on different types of computers.
(d) Advantages of Bus Topology:
(a) Requirement of short length cable.
(b) Economical, cost of communication media is minimum.
(c) It is flexible and it is quite easy to add new nodes or delete node from the network.
Disadvantages of Bus Topology:
(a) Difficulty in detection of faults because of no centralize control.
(b) Fault in the backbone cable can disrupt the functioning of the whole network.
Advantages of Star Topology:
(a) One device per connection.
(b) Easy to access.
Disadvantages of Star Topology:
(a) Long Length Cables.
(b) Dependency on central node.
(e) (i) Suitable topology is-STAR Topology
cbse-solved-papers-for-class-12-computer-science-c-paper-3-3
(ii) The server should be paced in wing A as it has the maximum number of computers. According to 80 : 20 rule also, server should be placed on A wing.
(iii) The Hub should be placed at each wing of network.
cbse-solved-papers-for-class-12-computer-science-c-paper-3-4
(iv) (a) Private cloud : Private cloud is a cloud infrastructure operated only for a single organization whether managed internally or by a third party & hosted internally or externally.
(b) Public cloud: A cloud is called a public cloud when the services vendered over a network is open for public use. This is not confined to a particular organization or business.

Class 8 Geography Chapter 3 Extra Questions and Answers Mineral and Power Resources

Class 8 Geography Chapter 3 Extra Questions and Answers Mineral and Power Resources

Join us as we traverse through the corridors of Earth’s hidden treasures, exploring the captivating journey of mineral and power resources. Let’s delve into the world of “Class 8 Geography Chapter 3 Extra Questions and Answers Mineral and Power Resources” and unravel the threads of economic prosperity, technological advancement, and the responsibility of responsible resource management. Read this also Extra Questions for Class 8 Social Science .

Class 8 Geography Chapter 3 Extra Questions and Answers Mineral and Power Resources

Mineral and Power Resources Class 8 Extra Questions and Answer Geography Chapter 3 Very Short Answers Type

Question 1.
What is a mineral?
Answer:
A naturally occurring substance that has a definite chemical composition is a mineral.

Question 2.
What is meant by a rock?
Answer:
A rock is an aggregate of one or more minerals but without definite composition of constituent of mineral.

Question 3.
Define open-cast mining.
Answer:
Minerals that lie at shallow depths are taken out by removing the surface layer; this is known as open- caste mining.

Question 4.
What does the term quarrying mean?
Answer:
Minerals that lie near the surface are simply dug out by the process known as quarrying.

Question 5.
Which country has no known mineral deposit in it?
Answer:
Switzerland has no known mineral deposit in it.

Question 6.
Name the two countries of Asia that have large iron ore deposits.
Answer:
China and India have large iron ore deposits.

Question 7.
How is salt obtained?
Answer:
Salt is obtained from seas, lakes and rocks.

Question 8.
Where are the oldest-rocks in world located in?
Answer:
The oldest rocks in the world are in western Australia.

Question 9.
Define Geothermal energy.
Answer:
Heat energy obtained from the earth is called geothermal energy.

Question 10.
What is Biogas?
Answer:
Organic waste such as dead plant and animal material and dung and kitchen waste can be converted into gaseous fuel called Biogas.

Question 11.
Name the greatest producer of Nuclear power.
Answer:
USA and Europe

Question 12.
Which type of energy is wind energy?
Answer:
Wind is an inexhaustible source of energy.

Question 13.
How are windmills used since times immemorial?
Answer:
Windmills have been used for grinding grain and lifting water since times immemorial.

Question 14.
Define Geothermal Energy.
Answer:
Heat energy obtained from Earth.

Question 15.
In which part of India there is a huge tidal mill farms?
Answer:
In the Gulf of Kachchh.

Question 16.
Where in India are the geothermal plants located?
Answer:
Manikaran in Himachal Pradesh and Puga Valley in Ladakh.

Question 17.
How is firewood widely used?
Answer:
It is widely used for cooking and heating.

Question 18.
What are the 2 main conventional sources of energy?
Answer:
Firewoods and fossil fuels.

Question 19.
Which is the most abundantly found fossil fuel?
Answer:
Coal is the most abundantly found fossil fuel.

Question 20.
What is thermal power?
Answer:
Electricity from coal is called thermal power.

Mineral and Power Resources Class 8 Extra Questions and Answer Geography Chapter 3 Short Answers Type

Question 1.
What are the uses of minerals?
Answer:
Minerals are used in many industries. Minerals which are used for gems are usually hard. These are then set in various styles of jewellery. Copper is another metal used in everything from coins to pipes, silicon used in computer industry is obtained from quartz. Aluminium obtained from its ore bauxite is used in automobiles and aeroplanes, bottling industry, buildings and even in kitchen cookware.

Question 2.
How is the distribution of iron placed in India?
Answer:
India has deposits of high-grade iron ore. The mineral is found mainly in Jharkhand, Odisha, Chhattisgarh, Madhya Pradesh, Goa, Maharashtra and Karnataka.

Question 3.
Name the major limestone producing states in India.
Answer:
Major limestone producing states in India are Bihar, Jharkhand, Odisha, Madhya Pradesh, Chhattisgarh, Rajasthan, Gujarat and Tamil Nadu.

Question 4.
What is coal and why is coal referred to as Buried Sunshine?
Answer:
Coal is the most abundantly found fossil fuel. It is used as a domestic fuel, in industries such as iron and steel, steam engines and to generate electricity. Electricity from coal is called Thermal Power.

The coal which we are using today was formed millions of years ago when giant ferns and swamps got buried under the layer of earth. Coal is therefore referred to as Buried Sunshine.

Question 5.
Define Tidal Energy and where was the first tidal energy station built?
Answer:
Energy generated from tides is called tidal energy. Tidal energy can be harnessed by building dams at narrow openings of the sea. During high tide the energy of the tides is used to turn the turbine installed in the dam to produce electricity. Russia, France and Gulf of Kachchh in India have huge tidal mill farms. The Ist tidal energy station was built in France.

Question 6.
How is petroleum found and what is petroleum and its derivatives known as and why?
Answer:
Petroleum is found between the layers of rocks and is derived from oil fields located in off shore and coastal areas. Then this is sent to refinery which processes the crude oil and produces a variety of products. Petroleum and its derivatives are called Black gold because they are valuable.

Mineral and Power Resources Class 8 Extra Questions and Answer Geography Chapter 3 Long Answers Type

Question 1.
Define mineral in brief and explain how they are formed without any human interference.
Answer:
A naturally occurring substance that has a definite chemical composition is a mineral. Minerals are not evenly distributed over space. They are concentrated in a particular area or rock formations. Some minerals are found in areas which are not easily accessible such as the Arctic ocean bed and Antarctica.

Minerals are formed in different types of geological environments, under varying conditions. They are created by natural processes without any human interference. They can be identified on the basis of their physical properties such as colour, density, hardness and chemical property such as solubility.

Question 2.
What is meant by Nuclear power? Explain the process how it is obtained. Also name the places of India where the nuclear power stations are located.
Answer:
Nuclear power is obtained from energy stored in the nuclei of atoms of naturally occurring radio active elements like uranium and thorium. These fuels undergo nuclear fission in nuclear reactors and emit power. The greatest producers of nuclear power are U.S.A and Europe. In India Rajasthan and Jharkhand have large deposits of Uranium.

Thorium is found in large quantities in the Monozite sands of Kerala. The nuclear power stations in India are located in Kalapakkam in Tamil Nadu, Tarapur in Maharashtra, Ranapratap Sagar near Kota in Rajasthan, Narora in U.P, and Kaiga in Karnataka.

Picture Based Questions Class 8 Geography Chapter 3 Mineral and Power Resources

Look at the picture given below and answer the following questions:
Class 8 Geography Chapter 3 Extra Questions and Answers Mineral and Power Resources img-1
Question:
1. What is shown in the figure?
2. Define drilling.
Answer:
1. This picture shows shore drilling of oil.
2. Deep wells bored to take petroleum and natural gas out is called drilling.

Map Skills Class 8 Geography Chapter 3 Mineral and Power Resources

Question 1.
On an outline Map of the World mark the following:
(i) Countries of Asia with large iron deposits.
(ii) The countries in Europe that are leading producer of iron-ore in the world.
Answer:
(i) China and India
(ii) Russia, Swedan, Ukraine, France.
Class 8 Geography Chapter 3 Extra Questions and Answers Mineral and Power Resources img-2

Question 2.
On an outline Map of India mark the following.
(i) Iron distribution in the states of India
(ii) Major Bauxite producing states.
Answer:
(i) Madhya Pradesh, Goa, Maharashtra and Karnataka.
(ii) Jharkhand, Odisha, Chhatisgarh, Madhya Pradesh, Gujarat, Maharashtra.
Class 8 Geography Chapter 3 Extra Questions and Answers Mineral and Power Resources img-3

Class 8 Geography Chapter 4 Extra Questions and Answers Agriculture

Class 8 Geography Chapter 4 Extra Questions and Answers Agriculture

Join us as we traverse through the corridors of the world’s farmlands, exploring the captivating journey of agriculture. Let’s delve into the world of “Class 8 Geography Chapter 4 Extra Questions and Answers Agriculture” and unravel the threads of sustenance, innovation, and the delicate balance between farming and nature. Read this also Extra Questions for Class 8 Social Science .

Class 8 Geography Chapter 4 Extra Questions and Answers Agriculture

Agriculture Class 8 Extra Questions and Answer Geography Chapter 4 Very Short Answers Type

Question 1.
What is arable land?
Answer:
The land on which the crops are grown is called arable land.

Question 2.
Mention the important inputs of agriculture.
Answer:
Seeds, fertilizers, machinery and labour.

Question 3.
Name the operations involved in agriculture.
Answer:
Ploughing, sowing, irrigation, weeding and harvesting.

Question 4.
Mention the outputs of agriculture.
Answer:
Crops, wool, dairy and poultry products.

Question 5.
On what basis can agriculture be classified.
Answer:

  • Geographical conditions,
  • demand of produce,
  • labour,
  • level of technology.

Question 6.
Name two major types of farming.
Answer:

  1. Subsistence farming
  2. Commercial farming.

Question 7.
What is subsistence farming?
Answer:
Subsistence farming is practised to meet the needs of the farmer’s family.

Question 8.
Into which two types can subsistence farming be classified?
Answer:

  1. Intensive subsistence,
  2. Primitive subsistence farming.

Question 9.
Name the crops cultivated in intensive subsistence agriculture.
Answer:
Wheat, maize, pulses and oilseeds.

Question 10.
Into which two types can primitive subsistence agriculture be divided?
Answer:

  1. Shifting cultivation,
  2. nomadic herding.

Question 11.
In which parts of the world is shifting cultivation practised?
Answer:

  • Amazon basin,
  • Tropical Africa,
  • Parts of South-East Asia,
  • North-East India.

Question 12.
Which types of farming are included in commercial farming?
Answer:

  • Commercial grain farming,
  • Mixed farming,
  • Plantation agriculture.

Question 13.
In the temperate grasslands of North America, why can only a single crop be grown?
Answer:
In the temperate grasslands of North America, only a single crop can be grown because severe winters- restrict the growing season.

Question 14.
Name two major plantations found in the tropical regions of the world.
Answer:

  1. Rubber plantation in Malaysia
  2. Coffee plantation in Brazil
  3. Tea plantation in India and Sri Lanka.

Question 15.
Name two important food crops, fibre crops and beverage crops.
Answer:

  1. Food crops – wheat,
  2. rice Fibre crops – jute,
  3. cotton Beverage crops – tea, coffee.

Question 16.
Name four leading producers of rice.
Answer:
China, India, Japan, Sri Lanka.

Question 17.
Name two places where two to three crops of rice are grown in a year.
Answer:

  1. West Bengal,
  2. Bangladesh.

Question 18.
During which season is wheat grown in India?
Answer:
Winters.

Question 19.
Name the leading producers of maize.
Answer:
North America, Brazil, China, Russia, Canada, India and Mexico.

Question 20.
In which countries is cotton grown as a major crop?
Answer:
China, USA, India, Pakistan, Brazil and Egypt.

Question 21.
Which countries produce best quality tea in the world?
Answer:
Kenya, India, China, Sri Lanka.

Question 22.
What are primary activities? Give examples.
Answer:
Primary activities include all those activities connected with extraction and production of natural resources. For example, agriculture, fishing, gathering etc.

Question 23.
What are secondary activities? Give examples.
Answer:
Secondary activities are concerned with the processing of natural resources. For example, manufacturing of steel, weaving of cloth etc.

Question 24.
What are tertiary activities? Give examples.
Answer:
Tertiary activities provide support to the primary and secondary sectors through services. For example, trade, banking, insurance etc.

Question 25.
In which parts of the world is nomadic herding practised?
Answer:
Nomadic herding is practised in the following regions –

  • Semi-arid and arid regions of Sahara.
  • Central Asia.
  • Rajasthan and Jammu and Kashmir in India.

Question 26.
Write the main features of commercial farming?
Answer:
Main features of commercial farming are:

  • Crops are grown and animals are reared for sale in the market.
  • The area cultivated and the amount of capital used is large.
  • Most of the work is done by machines.

Question 27.
What is mixed farming? In which parts of the world is it practised?
Answer:

  • In the mixed farming, the land is used for growing food and fodder crops and rearing livestock.
  • It is practised in Europe, Eastern USA, Argentina, Southeast Australia, New Zealand and South Africa.

Question 28.
Why do is development of a transport network essential for plantation farming?
Answer:
Development of a transport network is essential for plantation farming because the produce is processed on the farm itself or in nearby factories.

Question 29.
Mention the geographical requirements for the cultivation of rice.
Answer:

  • It is mainly grown in tropical and sub-tropical regions.
  • Rice needs high temperature, high humidity and rainfall.
  • It grows well in alluvial clayey soil.

Question 30.
Write the geographical requirements for wheat cultivation.
Answer:

  • Wheat requires moderate temperature and rainfall during growing season and bright sunshine at the time of harvest.
  • It thrives best in well-drained loamy soil.

Question 31.
Mention the geographical conditions required for maize cultivation.
Answer:

  • Maize requires moderate temperature.
  • It also needs well-drained fertile soils.

Question 32.
Write the geographical conditions required for cotton.
Answer:

  • Cotton requires high temperature, light rainfall, two hundred and ten frost-free days and bright sunshine.
  • It grows best on black and alluvial soils.

Question 33.
Mention the geographical conditions required for coffee cultivation.
Answer:

  • Coffee requires warm and wet climate and well-drained loamy soil.
  • Hill slopes are more suitable for the growth of this crop.

Question 34.
What is agricultural development? Also, write its ultimate aim.
Answer:

  • Agricultural development refers to the efforts made to increase farm production in order to meet the growing demand of increasing population.
  • Its ultimate aim is to increase food security.

Question 35.
How can farm production be increased?
Or
How can the ultimate aim of agricultural development be increased?
Answer:
Farm production can be increased in the following ways –

  • Increasing the cropped area.
  • The number of crops grown.
  • Improving irrigation facilities.
  • Use of fertilizers and high yielding variety of seeds.

Question 36.
On the outline map of India show the following places. Coffee producing state.
Answer:
Coffee producing state
Class 8 Geography Chapter 4 Extra Questions and Answers Agriculture img-1

Question 37.
What is sericulture? Name the factors influencing agriculture?
Answer:
Commercial rearing of silkworms is called sericulture. Factors influencing agriculture are as follows :

  • Favourable topography
  • Soil
  • Climate.

Question 38.
Write the main features of intensive subsistence agriculture.
Answer:

  • The farmer cultivates a small plot of land using simple tools and more labour.
  • Climate with large number of days with sunshine and fertile soil permit growing of more than one crop annually on the same field.
  • Major crops grown are rice, wheat, maize, pulses etc.
  • Intensive subsistence agriculture is practised in thickly populated areas of the monsoon regions of South, South-East and East Asia.

Question 39.
Write the main features of nomadic herding.
Answer:

  • Herdsmen move from place to place with their animals in search of fodder and water.
  • Herders have to move from one place to another due to climatic constraints and terrain.
  • Sheep, camel, cattle, yak and goats are the most commonly reared.
  • Animals provide milk, meat, wool, hides and other products to herders and their families.

Question 40.
Describe the regions where commercial grain farming is practised.
Answer:

  • Commercial grain farming is practised in the temperate grasslands of North America, Europe and Asia.
  • These regions are sparsely populated with large farms spreading over hundreds of hectares.
  • Due to severe winters, only a single crop can be grown.

Question 41.
Write a short note on millets.
Answer:

  • Millets are also known as coarse grains and can be grown on less fertile and sandy soils.
  • It is a hardy crop that needs low rainfall and high to moderate temperature and adequate rainfall.
  • Millets are grown in India, Nigeria, China and Niger.

Question 42.
Write the geographical condi¬tions required for the cultivation of tea.
Answer:

  • Tea requires cool climate and well-distributed high rainfall throughout the year for the growth of its tender leaves.
  • It needs well-drained loamy soils and gentle slopes.
  • Large number of labour is required to pick the leaves.

Multiple Choice Questions (MCQs)

1. ………………………….. is not a tertiary activity.
(a) Trade
(b) Manufacturing
(c) Transport
(d) Banking.
Answer:
(b) Manufacturing

2. Tick the odd pair in the following.
(a) Pisciculture – breeding of fish.
(b) Viticulture – cultivation of grapes.
(c) Horticulture – rearing livestock.
(d) Sericulture – rearing of silkworms.
Answer:
(c) Horticulture – rearing livestock.

3. Organic manure and natural pesticides are used instead of chemicals in this type of farming. What is it called?
(a) Subsistence farming
(b) Shifting cultivation
(c) Mixed farming
(d) Organic farming.
Answer:
(d) Organic farming.

4. Which of the following crops needs high temperature, high humidity, fertile soil and good rainfall?
(a) Rice
(b) Wheat
(b) Cotton
(d) Millets
Answer:
(a) Rice

5. Which of the following pairs is not correct?
(a) Tea – beverage
(b) Jute – Golden Fibre
(c) Wheat – corn
(d) Millets – coarse grains
Answer:
(c) Wheat – corn

Glossary:

→ Primary activities – They include activities connected with extraction and production of natural resources.

→ Secondary activities – These activities are concerned with the processing of natural resources.

→ Tertiary activities – These activities provide support to the primary and secondary sectors through services.

→ Sericulture – Commercial rearing of silkworms.

→ Viticulture – Cultivation of grapes.

→ Pisciculture – Breeding of fish in specially constructed tanks and ponds.

→ Horticulture – Growing vegetables, flowers and fruits for commercial use.

→ Agriculture – The science and art of cultivation on the soil, raising crops and rearing livestock.

→ Subsistence farming- This type of farming is practised to meet the needs of farmer’s family.

→ Nomadic herding – The herdsmen move from one place to another in search of fodder and water.

→ Commercial farming- The crops are grown and animals are reared for sale in the market.

→ Mixed farming – The land is used for growing food and fodder crops and rearing livestock.

→ Organic farming – The use of organic manure and natural pesticides instead of chemicals is known as organic farming.

→ Agricultural development – It refers to efforts made to increase farm production in order to meet the growing demand of increasing population.

Class 8 Geography Chapter 2 Extra Questions and Answers Land, Soil, Water, Natural Vegetation and Wildlife Resources

Class 8 Geography Chapter 2 Extra Questions and Answers Land, Soil, Water, Natural Vegetation and Wildlife Resources

Join us as we traverse through the corridors of Earth’s natural wonders, exploring the captivating journey of land, soil, water, natural vegetation, and wildlife resources. Let’s delve into the world of “Class 8 Geography Chapter 2 Extra Questions and Answers Land, Soil, Water, Natural Vegetation and Wildlife Resources” and unravel the threads of interconnectedness, conservation, and the harmony between humans and nature. Read this also Extra Questions for Class 8 Social Science .

Class 8 Geography Chapter 2 Extra Questions and Answers Land, Soil, Water, Natural Vegetation and Wildlife Resources

Land, Soil, Water, Natural Vegetation and Wildlife Resources Class 8 Extra Questions and Answer Geography Chapter 2 Very Short Answers Type

Question 1.
What is the full form of CITES?
Answer:
CITES is the Convention on International Trade in Endangered Species of Wild Fauna and Flora.

Question 2.
Define National Park?
Answer:
National Park is a natural area designated for the protection of ecological integrity of one or more ecosystem for the present and the future generation.

Question 3.
Name the classification of forests depending on when they shed their leaves?
Answer:
Forests are broadly classified as evergreen and deciduous depending on when they shed their leaves.

Question 4.
Primarily on what factors does the growth of vegetation depends upon?
Answer:
The growth of vegetation depends primary on temperature and moisture.

Question 5.
Why is river Yamuna getting polluted?
Answer:
River Yamuna is getting polluted due to sewage, industrial effluents and garbage released into it.

Question 6.
What quantity of water does a dripping tap waste in a year?
Answer:
A dripping tap water 1200 litres of water in a year.

Question 7.
What are the two major threats to soil as a resource?
Answer:
Soil erosion and depletion are the major threats to soil as a resource.

Question 8.
How much time period is required for the formation of just one centimetre of soil?
Answer:
It takes hundreds of years to make just one centimetre of soil.

Question 9.
Which natural resource covers only about 30% of the total area of the earth’s surface?
Answer:
Land

Question 10.
What is the main reason behind the uneven distribution of population in different parts of the world?
Answer:
It is mainly due to varied characteristics of land and climate.

Question 11.
On what basis land is classified into private land and community land?
Answer:
On the basis of ownership.

Question 12.
What has led to large scale destruction of forest cover and arable land?
Answer:
Growing population and their ever growing demand have led to a large scale destruction of forest cover and arable land.

Question 13.
Why is Hazard Mapping done?
Answer:
Hazard mapping is done to locate areas prone to landslide.

Question 14.
What determines the type of soil?
Answer:
Landform determines the type of soil.

Question 15.
In which year does the consumption of water for human use was 3850 cai/year?
Answer:
In 1975.

Question 16.
Why is Earth appropriately called water planet?
Answer:
Because 3/4th of the earth’s surface is covered with water.

Question 17.
What percent of fresh water is present on Earth?
Answer:
Only about 2.7%.

Question 18.
What is the major problem faced by the world today?
Answer:
Access to clean and adequate water sources is a major problem faced by the world today.

Question 19.
On what does the growth of vegetation depends primarily?
Answer:
The growth of vegetation depends primarily on temperature and moisture.

Question 20.
Give some examples of species protected under CITES.
Answer:
Bears, dolphins, cacti, corals, orchids and aloes, etc.

Land, Soil, Water, Natural Vegetation and Wildlife Resources Class 8 Extra Questions and Answer Geography Chapter 2 Short Answers Type

Question 1.
What is soil and how is soil made fertile?
Answer:
The thin layer of grainy substance covering the surface of the earth is called soil. It is closely linked to land. Landforms determine the type of soil. Soil is made up of organic matter, minerals and weathered rocks found on the earth. This happens through the process of weathering. The right mix of minerals and organic matter make the soil fertile.

Question 2.
What does the weathering mean and how does the weathering help soil?
Answer:
Weathering is the breaking up and decay of exposed rocks by temperature changes, frost action, plants, animals and human activity and soil is made up of organic matter, minerals and weathered rocks found on the earth. This happens through the process of weathering.

Question 3.
How does the major factors of soil formation play an important role?
Answer:
The major factors of soil formation are the nature of the parent rock and climatic factors and

  1. Parent Rock: Determines colour, texture, chemical properties minerals, content, permeability.
  2. Climate: Temperature, rainfall influence rate of weathering and humus.
  3. Relief: Altitude and slope, determine accumulation of soil.
  4. Flora, Fauna and Microorganism: Affect the rate of humus formation.
  5. Time: Determines thickness of soil profile.

Question 4.
What are some broad mitigation techniques of landslide?
Answer:
Some broad mitigation techniques of landslide are as follows:

  1. Hazard mapping for locating lanslides prone area, so that building settlements can be avoided.
  2. Construction of retention wall to stop land from slipping.
  3. Increase in the vegetation cover to arrest landslide.
  4. The surface drainage control works to control the movement of landslide along with rain water and spring flows.

Question 5.
How is the land classified on the basis of ownership? What is the concept of common property resources?
Answer:
Land can be classified on the basis of ownership as private and community land. Private land is owned by the individuals whereas, community land is owned by the community for common uses like collection of fodder, fruits, nuts or medicinal herbs. These community lands are called common property resources.

Land, Soil, Water, Natural Vegetation and Wildlife Resources Class 8 Extra Questions and Answer Geography Chapter 2 Long Answers Type

Question 1.
What is a land and what is the concept of Land use?
Answer:
Land is among the most important natural resources. It covers only about thirty per cent of the total area of the earths’s surface and all parts of this small percentage are not habitable.

Land Life
Land is used for different purposes such as agriculture, forestry, mining, building houses, roads and setting up of industries. This is commonly termed as Land use. The use of land is determined by the physical factors such as topography, soil, climate, minerals and availability of water. Human factors such as population and technology are also important determinants of land use pattern.

Question 2.
Describe Landslide and Mitigation mechanism in brief.
Answer:
Landslides are simply defined as the mass movement of rock debris or earth down a slope. They often take place in conjunction with earthquakes, floods and volcanoes. A prolonged spell of rainfall can cause heavy landslide that can block the flow of river for quite some time.

The formation of river blocks can cause havoc to the settlements downstream on its bursting. In the hilly terrain landslides have been a major and widely spread natural disaster that often strike life and property and occupy a position of major concern.

Mitigation mechanism is the advancement in scientific techniques which has empowered us to understand what factors cause landslides and how to manage them.

Picture Based Questions Class 8 Geography Chapter 2 Land, Soil, Water, Natural Vegetation and Wildlife Resources

Look at the picture given below and answer the following questions:
Class 8 Geography Chapter 2 Extra Questions and Answers Land, Soil, Water, Natural Vegetation and Wildlife Resources img-1
1. What does the picture show.
2. Under which category is Tsunami placed?
Answer:
1. The picture shows the loss of rainforest in Great Nicobar after Tsunami.
2. Natural factors accelerating the process of extinction of resources.

Map Skills Class 8 Geography Chapter 2 Land, Soil, Water, Natural Vegetation and Wildlife Resources

Question 1.
On an outline map of India mark the following:
(i) Kaziranga National Park
(ii) Himachal Pradesh
(iii) Great Nicobar
Answer:
Class 8 Geography Chapter 2 Extra Questions and Answers Land, Soil, Water, Natural Vegetation and Wildlife Resources img-2

Question 2.
On an outline Map of the world mark the regions of the world where there is scarcity of water.
Answer:
Class 8 Geography Chapter 2 Extra Questions and Answers Land, Soil, Water, Natural Vegetation and Wildlife Resources img-3

Class 8 Geography Chapter 1 Extra Questions and Answers Resources

Class 8 Geography Chapter 1 Extra Questions and Answers Resources

In this chapter, “Class 8 Geography Chapter 1 Extra Questions and Answers Resources,” we’ll traverse through the abundant reserves of nature, discovering the invaluable gifts bestowed upon us. From the riches beneath the earth’s surface to the boundless expanse of the atmosphere, from the life-sustaining water bodies to the fertile lands, this narrative unveils the wealth of resources that shape our existence. Read this also Extra Questions for Class 8 Social Science .

Class 8 Geography Chapter 1 Extra Questions and Answers Resources

Resources Class 8 Extra Questions and Answer Geography Chapter 1 Very Short Answers Type

Question 1.
Define localized resources.
Answer:
The resources which are found only at certain places are localized resources.

Question 2.
What is meant by natural resources?
Answer:
Resources that are drawn from nature and used without much modifications are called natural resources.

Question 3.
Give some examples of abiotic resources.
Answer:
Soil, rocks, minerals, etc. are few examples of abiotic resources.

Question 4.
How are the resources generally classified?
Answer:
Resources are generally classified into different groups depending upon their level of development and use; origin; stock and distribution.

Question 5.
What do you understand by the term sustainable development?
Answer:
It means balancing the need to use resources and also conserve them for the future purpose.

Question 6.
Define Patent.
Answer:
Patent means the exclusive right over any idea or invention.

Question 7.
What is the shortest meaning of ‘value’?
Answer:
Value means worth

Question 8.
Give an example of localized resources.
Answer:
An example is iron ore.

Question 9.
What acts as an important factor changing a substance into resources?
Answer:
Technology acts as an important factor.

Question 10.
Define resource conservation.
Answer:
It is defined as using resources carefully and giving them time to get renewed.

Question 11.
Define Technology.
Answer:
It is the application of latest knowledge and skill in doing or making things.

Question 12.
Which of the resources category has a limited stock?
Answer:
Non-renewable resource of energy has a limited stock.

Question 13.
Define the term utility.
Answer:
The state of being useful, profitable or beneficial is termed as utility.

Question 14.
What is meant by Stock of Resources?
Answer:
It is the amount of resources available for us.

Question 15.
Name the two groups of resource whose classification is done based on development and its use.
Answer:

  1. Actual resources
  2. Potential resources

Question 16.
Give examples of some human-made resources.
Answer:
Building, roads, bridges, machinery and vehicles, etc….,

Resources Class 8 Extra Questions and Answer Geography Chapter 1 Short Answers Type

Question 1.
Describe the term Patent and Technology.
Answer:
The term patent and technology is described as:

  1. Patent is meant by the exclusive right over any idea or invention.
  2. Technology is the application of latest knowledge and skill in doing or making things.

Question 2.
What are the renewable resources and how can their stock of certain resource get affected?
Answer:
Renewable resources are those which get renewed Or replenished quickly. Some of these are unlimited and are not affected by human activities, such as solar and wind energy. Yet careless use of certain renewable resources like water, soil and forest can affect their stock.

Question 3.
What are the Human Made Resources?
Answer:
Natural substances become resources only when their original form has been changed. Iron ore was not a resource until people learnt to extract iron from it. People use natural resources to make buildings, bridges, roads, machinery and vehicles, that are known as human made resources, even the technology is defined as a human made resource.

Question 4.
What do you know about Human Resource?
Answer:
Human resource refers to the number (quantity) and abilities (mental and physical) of the people. Though, there are differing views regarding treatment of humans as a resource, one cannot deny the fact that it is the skill of human that helps in transferring the physical material into a valuable resource.

Question 5.
Define what is meant by the Actual Resources?
Answer:
Actual resources are the resources whose quantity and quality are known and these resources are being used in the present. The rich deposits of coal in Ruhr region of Germany and petroleum in West Asia, the dark soils of the Deccan plateau in Maharashtra are all actual resources.

Resources Class 8 Extra Questions and Answer Geography Chapter 1 Long Answers Type

Question 1.
Describe the following terms in short.
(i) Actual resources
(ii) Non-renewable resources
(iii) Ubiquitous resources
(iv) Natural resources
Answer:

  • Actual resources: These are those resources whose quantity is known. These resources are being used in the present.
  • Non-renewable resources: These are those resources which have a limited stock. Ohce the stock are exhausted it may take thousands of years to be renewed and replenished.
  • Ubiquitous resources: Resources that are found everywhere are Ubiquitous resources.
  • Natural resources: Resources that are drawn from nature and used without much modification are called Natural resources.

Question 2.
Define the following.
(i) Human-made resources
(ii) Human resources
(iii) Sustainable development
Answer:
(i) Human-made resources: Natural substances converted into resources by changing their original form. For example, iron ore was not a resource until people learnt to extract iron from it. People use natural resources to make buildings, bridges, roads, machinery and vehicles, which are known as human-made resources. Technology is also a human-made resource.

(ii) Human resources: People can make the best use of nature to create more resources when they have the knowledge, skill and technology to do so. That is why human beings are a special resource. People are human resources.

(iii) Sustainable development: It means careful utilisation of resources so that besides meeting the requirements of the present, it also takes care of future generations.

Picture Based Questions Class 8 Geography Chapter 1 Resources

Look at the picture given below and answer the following questions:
Class 8 Geography Chapter 1 Extra Questions and Answers Resources img-1
1. Under which resource category the above pic has been categorized?
2. How do wind fans generate energy?
Answer:
1. Under the category of Potential Resources.
2. Wind fans generate energy using wind cycles.

Class 8 History Chapter 9 Extra Questions and Answers Women, Caste and Reform

Class 8 History Chapter 9 Extra Questions and Answers Women, Caste and Reform

Step into the realms of social transformation and women’s emancipation as we embark on an enlightening journey through Class 8 History Chapter 9 Extra Questions and Answers Women, Caste and Reform. Titled “Women, Caste, and Reform,” this chapter delves into the fascinating and tumultuous history of women’s rights and caste-based reforms in India. Read this also Extra Questions for Class 8 Social Science .

Class 8 History Chapter 9 Extra Questions and Answers Women, Caste and Reform

Women, Caste and Reform Class 8 Extra Questions and Answer History Chapter 9 Very Short Answers Type

Question 1.
What was Brahmo Sabha known as later?
Answer:
The later known name of Brahmo Sabha was the Brahmo Samaj.

Question 2.
Why were the people such as Rammohan Roy described as reformers?
Answer:
They were described as reformers because they felt that the best way to ensure such changes was by persuading people to give up old practices and adopt a new way of life.

Question 3.
Name the practice against which Rammohan Roy campaigned.
Answer:
He began campaigning against the practice of Sati.

Question 4.
Who amongst the reformers was the most famous and used ancient texts to suggest that widows could remarry?
Answer:
Ishwar Chandra Vidyasagar used the ancient texts to suggest that widows could remarry.

Question 5.
Who founded Arya Samaj?
Answer:
Swami Dayanand Saraswati founded Arya Samaj.

Question 6.
What was criticized in the book, Stripurushtulna written by Tarabai Shinde?
Answer:
The book criticized the social differences between men and women.

Question 7.
What did the nationalist’s leaders promise to women?
Answer:
Nationalist leaders promised that there would be full suffrage for all men and women after independence.

Question 8.
For which purpose was the Paramhans Mandali founded in 1840?
Answer:
It was founded to work for the abolition of caste.

Question 9.
Where did the number of Mahar people found jobs?
Answer:
A number of Mahar people found jobs in the Mahar Regiment.

Question 10.
Who wrote the book named Gulamgirit
Answer:
Jyotirao Phule wrote the book named Gulamgiri.

Question 11.
What does the social reformers felt?
Answer:
They felt some changes are essential in society and unjust practices needed to be sorted out.

Question 12.
Define ‘Sati’.
Answer:
Widows who used to burn themselves in the funeral of their husbands were known as Sati.

Question 13.
Who was Raja Ram Roy?
Answer:
He was a great social reformer.

Question 14.
Who was the founder of Brahmo Samaj?
Answer:
Raja Ram Mohan Roy.

Question 15.
In which year Brahmo Samaj was founded?
Answer:
In 1830.

Question 16.
Name the founder of Ramakrishnan Mission.
Answer:
Swami Vivekananda.

Question 17.
When was the Prarthana samaj established?
Answer:
In 1867.

Question 18.
At what place Veda Samsy was established?
Answer:
It was established in Madras. (Chennai).

Question 19.
Where does the Madigas belongs to?
Answer:
Madigas belongs to Andhra Pradesh.

Question 20.
What was John Allen?
Answer:
John Allen was the coolie ship.

Question 21.
What work was performed by Madigas?
Answer:
Madiga were experts at cleaning hides, tanning them for use, and sewing sandals.

Question 22.
Who was Pandita Rama Bai?
Answer:
Pandita Rama Bai was a great scholar of Sanskrit.

Women, Caste and Reform Class 8 Extra Questions and Answer History Chapter 9 Short Answers Type

Question 1.
What was the childhood experience of Dr Ambedkar because of his belonging to the Mahar Community?
Answer:
Dr Ambedkar was bom into a Mahar family. As a child he experienced what caste prejudice meant in everyday life. In school he was forced to sit outside the classroom on the ground. He was not even allowed to drink water from taps that upper-caste children used.

Question 2.
What was the argument of E.V. Ramaswamy Naicker about the untouchables?
Answer:
E.V. Ramaswamy argued that untouchables were the upholders of an original Tamil and Dravidian culture which had been subjugated by BrahmAnswer: He felt that all religious authorities saw social divisions and inequality as God-given. Untouchables had to free themselves from all religions in order to achieve social equality.

Question 3.
What was the objective of associations founded by the upper caste Hinduism?
Answer:
The objectives of these associations were to uphold caste distinctions as a cornerstone of Hinduism and how things were sanctified by scriptures.

Question 4.
What was claimed by Phule about the time period before the Aryan rule?
Answer:
Phule claimed that before the Aryan rule there existed a golden age when warrior peasants tilled the land and ruled the Maratha countryside in just and fairways. He proposed that Shudras and Ati Shudras should unite to challenge caste discrimination.

Question 5.
How did Muslim women play a notable role in promoting education among women?
Answer:
Muslim women like the Begums of Bhopal played a notable role in promoting education among women. They founded primary school for girls at Aligarh. Another remarkable woman named Rokeya Sakhawat Hossain started schools for Muslim girls in Patna and Calcutta. She also argued that religious leaders of every faith accorded an inferior place to women.

Women, Caste and Reform Class 8 Extra Questions and Answer History Chapter 9 Long Answers Type

Question 1.
Explain the Singh Sabha Movement in brief.
Answer:
The Singh Sabha Movement is a reform organization of the Sikhs, the First Singh Sabhas were founded at Amritsar in 1873 and at Lahore in 1879. The Sabhas aimed to rid Sikhism of superstitions, caste distinctions and practices seen by them as non-Sikh. They promoted education among the Sikhs, often combining modern instruction with Sikh teachings.

Question 2.
How challenging was the life for Dr B.R. Ambedkar during his childhood when he experienced caste prejudice in everyday life? How did he challenge the problems faced by low caste people?
Answer:
Ambedkar was born into a Mahar family. As a child he experienced what caste prejudice meant in everyday life. In school he was forced to sit outside the classroom on the ground and was not allowed to drink water from taps that upper caste children used.

After finishing school, he got a fellowship to go to the U.S. for higher studies. On his return to India in 1919, he wrote extensively about upper caste power in contemporary society.

In 1927 Ambedkar started a temple entry movement. The movement of temple entry was participated by Mahar caste followers. Brahman priests were outraged when the Dalits used water from the temple tank. Ambedkar led 3 such movements for temple entry between 1927 and 1935. His aim was to make everyone see the power of caste prejudices within the society.

Picture Based Questions Class 8 History Chapter 9 Women, Caste and Reform

Look at the picture given below and answer the following questions:
1.
Class 8 History Chapter 9 Extra Questions and Answers Women, Caste and Reform img-1
(i) Name the following and was drawn by whom?
(ii) Who were not allowed to enter the temples?
Answer:
(i) The gateway to the Madurai temple, drawn by Thomas Daniell.
(ii) untouchables.

2.
Class 8 History Chapter 9 Extra Questions and Answers Women, Caste and Reform img-2
(i) What has been shown in the picture?
(ii) When was it established?
(iii) by whom it was established?
Answer:
(i) The Khalsa College Amritsar.
(ii) In 1892.
(iii) Leaders of the Singh Sabha movement